FREE Practice MCAT

Page 1

MCAT Practice Test 1.pages

Part I of IV Chemical and Physical Foundations of Biological Systems 59 Questions (45 Passage Based Questions + 14 Discrete Questions) 95 Minutes

1


MCAT Practice Test 1.pages

The mechanism by which the heart reliably pumps blood to the body can be understood by looking solely at the mechanics of the right ventricle, the thickest chamber of the heart. From a fluid motion perspective, the right ventricle can be modelled as a single-chamber pump (Figure 1) attached in series to two pressure-activated, one-way valves. Using radiometric techniques, the volume and shape of the heart can be imaged in real time, as can the volume of blood that passes through the heart during each part of the cardiac cycle. This data can be combined to generate a pressurevolume diagram for the left-ventricle, corresponding to a single, full cardiac cycle (Figure 2). During the diastolic phase (segment AB in the Figures), the heart refills with blood from its contracted state at the end of the previous cycle. Thus, this phase is essentially a return stroke, in which blood slowly fills the heart, increasing its volume without changing the pressure due to the flexibility of the walls. The mitral valve closes at B, at which point the heart begins to contract around the trapped blood. The aortic valve opens at C, but the heart continues to contract---resulting in a rapid efflux of blood at a constant pressure. The aortic valve closes at D, and the heart tissue expands, relieving pressure, until the mitral valve re-opens at A, blood rushes back in, and the cycle repeats. Figure 1: A simple diagram of the four phases of the cardiac cycle

2


MCAT Practice Test 1.pages

Figure 2: A pressure-volume diagram for the four phases of the cardiac cycle

1. Which of these following reasons best describes why the volume remains constant despite pressure increase during section BC? Please choose from one of the following options. 1 Liquid incompressibility 2 Blood efflux 3 Boyle’s law 4 The ideal gas law

3


MCAT Practice Test 1.pages

2. During the pressurization phase BC, what is the pressure in the narrow valve entrance compared to the wider chamber? Please choose from one of the following options. 1 Smaller 2 Larger 3 The same 4 Zero 3. During the efflux phase CD, what is the pressure in the narrower valve entrance compared to the wider chamber? Please choose from one of the following options. 1 Equal 2 Larger 3 Smaller 4 Zero 4. During which segment of the cycle does the mechanical potential energy of the heart increase? Please choose from one of the following options. 1 CD 2 AB 3 BC 4 DA 5. Which of the following quantities is given by the area enclosed by the loop? Please choose from one of the following options. 1 The total number of moles of blood in the heart times the temperature 2 The net force exerted during one cardiac cycle 3 The total tissue displacement during one cardiac cycle 4 The total work performed during one cardiac cycle

4


MCAT Practice Test 1.pages

Muons are elementary, charged particles that undergo the same interactions as electrons, but which have much greater masses. For this reason, muons could be used as an alternative to electrons in transmission microscopy, in which the pattern formed by scattered, charged particles after passing through a sample may be used to infer the structure of biological tissues. Such methods are useful for probing structural details of biological materials that are smaller than the optical diffraction limit, which determines the smallest feature sizes that can be observed using a traditional, light-based microscope. A simple muon detector (Figure 1) consists of an array of diodes. Each diode emits an electric signal when a muon passes through it, and so the specific diode in the array that emits a signal after a muon strike indicates the two-dimensional location of the strike. A beam of muons with known energy passes through the sample, and at each beam location, the location on the detector plate at which muons arrive after passing through the sample is recorded. Locations at which the beam was most strongly deflected indicate the presence of internal structures, resulting in a twodimensional image of the tissue. The total deflection of the incident beam by the sample is, at most, a few degrees for the densest parts of the sample. This technique has recently been generalized to provide three-dimensional information about samples in a method known as muon tomography. In order to gain information about the three-dimensional scattering field with which the muon interacts before it reaches the detector, the direction that each muon is traveling after it exits the tissue sample must be known. This method requires at least two detector arrays, since the velocity vector of the muon can be calculated from the 2D location on each detector array where the muon struck, the distance between the two arrays, and the time between the two detection events. Assume that the muons are heavy enough to be treated as point masses subject to classical electromagnetism and Newton’s laws. Additionally, assume that the detectors themselves barely affect the muons’ trajectories.

5


MCAT Practice Test 1.pages

Figure 1: The elements of a muon measurement assembly. Muons exit the source and are deflected as they pass through the sample. The deflected muons pass through one of the diodes in the diode array, which indicates their position in the plane of the array.

6


MCAT Practice Test 1.pages

6. Which of the following best describes the cause of the small-angle deflections of the incident muon beam? Please choose from one of the following options. 1 The muons are captured by unfilled valence orbitals within the tissue, and then re-emitted. 2 The muons exchange their spin with the molecules in the tissue, causing their trajectories to become curved due to exchange of angular momentum. 3 The muons are repelled by the strong nuclear force arising in the nuclei of the tissue. 4 The muons are electromagnetically repelled by the electrons in the tissue. 7. Which of the following would DECREASE the resolution of the three-dimensional structure of a tissue sample in the transmission microscope? Please choose from one of the following options. 1 Covering the sample in a conductive coating or foil. 2 Applying the beam from multiple incident angles and recording changes in the scattering field. 3 Slicing the sample into thin slices, and imaging each slice separately. 4 Increasing the energy of the beam of muons directed at the sample. 8. In order to reconstruct the three-dimensional structure of a tissue during muon tomography, which of the following quantities must be measured during the experiment? I. The total charges of the atoms within the tissue sample II. The velocity of muons entering the sample III. The velocity of muons exiting the sample Please choose from one of the following options. 1 II and III 2 I and III 3 I and II 4 I, II, and III

7


MCAT Practice Test 1.pages

9. If muons are travelling towards the first detector along a direction normal to the plane of the diode array, where should a second detector array be placed so as to allow the most precise determination of the three-dimensional structure of the sample? Please choose from one of the following options.

1

2

3

4

8


MCAT Practice Test 1.pages

10. In practice, muon-based measurements are rarely used on living tissues. Which of the following would NOT be a problem with using muons instead of electrons? Please choose from one of the following options. 1 Muons are more difficult to generate 2 Muons are more likely to lose their charge as they pass through the sample 3 Muons require a greater beam energy to achieve a given incident momentum 4 Muons are heavier and thus decay faster

9


MCAT Practice Test 1.pages

During MRI, a magnetic field is used to align the spins of protons, which have an intrinsic magnetic moment coupled to the direction of their spins. As a result of this static field aligning the individual proton spins and magnetic moments, the sample acquires a net, measurable magnetization parallel to the applied field. Once the protons are aligned with the static field, radio-frequency (RF) photons are then applied to the protons, which cause them to enter a higher energy state with their spins aligned antiparallel to the field. The steps are outlined in Figure 1. During measurement, the applied RF field is then removed, and the protons gradually return to the lower-energy, aligned state, emitting photons with the same energy as the exciting pulse. This relaxation occurs gradually, with the number of spins aligned opposite the static field decreasing exponentially in time with a time constant known as T1, which depends on composition and structure of the tissue. In addition, neighboring spins in the sample tend to become aligned with one another due to pairwise magnetic interactions, which further improves the MRI signal. After the removal of the RF, this spin-spin alignment exponentially decays with a different time constant,T2, which also depends on material properties. The dependence of T1 and T2 on structural properties gives rise to contrast between different tissues in MRI. In many applications, “contrast agents� consisting of fluids with particularly short T1 times are used in order to differentiate otherwise similar tissues. Experimental results indicate a relationship between the observed relaxation timescale and the concentration of contrast agent used, as shown in Figure 2. Figure 1: The alignment of proton spins before the measurement begins, after application of a static magnetic field, and after application of a radiofrequency (RF) field that excites them to a higher energy state aligned opposite the field.

10


MCAT Practice Test 1.pages

Figure 2: The strength of the MRI signal along two, perpendicular measurement axes allows T1and T2 to be separately observed for different concentrations of contrast agent. A: Measurement of the signal along one direction allows T1to be observed for two different concentrations of contrast agent X. B: Measurement along another, orthogonal direction allows T2 to be observed for different concentrations of contrast agent Y.

Data adapted from: Yoo, B., & Pagel, M. D. (2007). An overview of responsive MRI contrast agents for molecular imaging. Frontiers in Bioscience: a Journal and Virtual Library, 13, 1733-1752. 11


MCAT Practice Test 1.pages

11. Based on the description of MRI found in the passage, which of the following is most likely proportional to the “signal� graphed against time in Figure 2A? Please choose from one of the following options. 1 The number of RF photons emitted by different parts of the sample. 2 The net magnetization of the sample along the direction of the applied RF field. 3 The number of RF photons absorbed by the sample during excitation. 4 The net magnetization of the sample along the direction of the static magnetic field. 12. Suppose that the applied RF field also heats the sample slightly. What effect would this have on the apparent T1 observed after the RF is removed? Please choose from one of the following options. 1 It would depend on the chemical composition of the contrast agent 2 T1 would remain the same 3 T1 decreases 4 T1 increases 13. Which of the following explanations provides the best physical reasoning for why increased concentration of a contrast agent would change the effective T2 of the contrast agent? Please choose from one of the following options. 1 A reduced mean distance between spins increases their electromagnetic coupling. 2 A higher density of emitted photons results in greater re-absorption of photons by neighboring spins. 3 A higher density of photons emitted during relaxation provokes stimulated emission of photons from nearby protons. 4 Higher concentrations of contrast agents shield particles from the applied magnetic field.

12


MCAT Practice Test 1.pages

14. In the Figure, which of the tested agents displays the largest and which displays the largestT2? 1 2 3 4

T1,

Largest T1: High X and Largest T2: Low Y Largest T1: Low X and Largest T2: Low Y Largest T1: High X and Largest T2: High Y Largest T1: Low X and Largest T2: High Y

15. The frequency of the applied RF signal used to excite spins is directly proportional to the magnitude of the static magnetic field used to align the spins, with proportionality constant 5 hz/T. If the strength of the applied field is known to be 20 T plus or minus 3 T, which of the following correctly describes the uncertainty in the INVERSE frequency (1/frequency)? Please choose from one of the following options. 1 2 3 4

4s 3/5s 3 / 2000 s 1 / 15 s

13


MCAT Practice Test 1.pages

Questions 16. 17. 18. 19. 20. are NOT based on passages. 16. Which of the following vector combinations will result in the least amount of displacement? (Note: Vectors a, b, d, and E have magnitudes double that of vectors c and f.)

Please choose from one of the following options. 1 2 3 4

14


MCAT Practice Test 1.pages

17. A car travels from point A to B in 3 hours and returns back to point A in 5 hours. Points A and B are 150 miles apart along a straight highway. Which of the following statements most accurately describes the motion of the car? Please choose from one of the following options. 1 The average velocity is 40 miles per hour. 2 The average speed is 37.5 miles per hour. 3 The car travels at 50 mph for the first half and 30 mph for the second half. 4 The total displacement of the trip is 300 miles. 18. On which of the following does the buoyant force on an object floating on the surface of a liquid most directly depend? Please choose from one of the following options. 1 viscosity of the liquid 2 density of the liquid 3 mass of the liquid 4 volume of the object 19. After MgCl2 dissolves in a neutrally charged solvent, what is the net charge of the solution? 1 2 3 4

The solution becomes negatively charged due to the majority Cl- ions The solution becomes positively charged due to the stronger Mg+2 ions The solution remains neutrally charged. The solution becomes positive or negative, depending on the relative Mg+2 and Cl- concentrations

15


MCAT Practice Test 1.pages

20.Which statement below is true? Please choose from one of the following options. 1 Sound waves are longitudinal waves and they propagate parallel to the transmitting medium. 2 Sound waves can propagate as longitudinal or transverse waves, depending on the temperature. 3 Sound waves can propagate as longitudinal or transverse waves, depending on the transmitting medium. 4 Sound waves are transverse waves and they propagate perpendicular to the transmitting medium.

16


MCAT Practice Test 1.pages

An electrocardiogram (ECG) measures the heart’s electrical activity using a series of probes placed on the skin. In addition to detecting rhythmic electrical activity due to the heart’s beating, an ECG can detect other processes, like spasms, that occur with different frequencies than the heartbeat. The ECG is able to convert data indicating the magnitude of the electrical activity as a function of time into information about specific processes that occur in the heart over different timescales using a mathematical operation known as a Fourier transform. In a Fourier transform, a time-varying signal is converted into a histogram showing the various frequencies that constitute the signal. As a result, an ECG voltage versus time graph can be replotted as an amplitude versus frequency graph, which indicates the relative presence of various frequencies in the original voltage reading. For example, The Fourier transform of a time series showing an undulating sine wave with frequency 1 hz would be a histogram with peaks at ±1 hz (Figure 1). For a more complex signal consisting of many different sine waves superimposed, the Fourier transform shows the relative amounts of various frequencies present in the signal. For example, in the Figure 2, the Fourier transform of the voltage signal

V(t)=(1 / 3 ) sin(2 π 3 t) + 1sin(2 π t) (Figure 2A) produces a bar chart with a higher peak at1 hz than at3 hz (Figure 2B). A mathematical theorem guarantees that a valid Fourier transform can be generated for any periodic signal, which equivalently suggest that any periodic ECG signal can be represented as a sum of sine waves with various frequencies and amplitudes. The raw output of an ECG is a periodic signal indicating the electrical activity arising from a patient’s heart as a function of time, but the shape of this signal is generally NOT purely sinusoidal-instead, it consists of a superposition of many different periodic processes in the body which generally ARE sinusoidal. As a result, the Fourier transform of an ECG signal can be used to indicate the presence or absence of various processes that occur with different characteristic frequencies based on their presence or absence in the Fourier histogram. 17


MCAT Practice Test 1.pages

Figure 1: A) A sinusoidal output from an ECG and B) its Fourier transform.

Figure 2: A) An ECG signal consisting of two superimposed sinusoids and B) its Fourier transform.

18


MCAT Practice Test 1.pages

Figure 3: The output of an ECG test for a healthy patient contains many different frequencies and, as a result, is generally not a single sine wave.

19


MCAT Practice Test 1.pages

21. Suppose a rare genetic mutation doubles the diameters of the electrically conducting cells that transmit electrical activity throughout the heart and body. Which of the following describes the most direct effect this would have on observed ECG voltage fluctuations if all other aspects of the patient (height, weight, composition) remain the same? Please choose from one of the following options. 1 A doubling of the amplitude. 2 A doubling of the frequency. 3 No direct effect. 4 A quadrupling of the amplitude.

20


MCAT Practice Test 1.pages

22. Which of the following traits MUST be present in a Fourier transform of Figure 3? Please choose from one of the following options. 1 A peak at 2 hz 2 A peak with amplitude 1 V 3 A peak with amplitude 2 V 4 A peak at .5 hz

21


MCAT Practice Test 1.pages

23. Which of the following graphs most likely represents the Fourier transform of an ECG reading that is well-fitted by the function

V(t) = 4 sin(2Ď€ 4 t )+ sin(2Ď€ 8 t ) Please choose from one of the following options.

1

22


MCAT Practice Test 1.pages

2

3

23


MCAT Practice Test 1.pages

4 24. Which of the following most directly determines the MINIMUM frequency that an ECG device can measure? Please choose from one of the following options. 1 The minimum change in voltage that the device can detect 2 The minimum total voltage that the device can detect. 3 The total time for which the voltage is measured 4 The minimum time interval between two measurement points on the voltage versus time graph. 25. Which of the following most directly determines the MAXIMUM frequency that an ECG device can measure? Please choose from one of the following options. 1 The minimum change in voltage that the device can detect 2 The minimum time interval between two measurement points on the voltage versus time graph. 3 The minimum total voltage that the device can detect. 4 The total time for which the voltage is measured

24


MCAT Practice Test 1.pages

A simple model of human standing is given by an inverted pendulum, a one-dimensional, classical model for the motion of a single, massive particle (Figure 1). A standing human is approximated as a single mass (located at the center of mass) separated from the ground by a massless rod of fixed length L. The feet are treated as attached to the ground by a fulcrum, such that the center of mass can only undergo motion along an arc of radius L around the feet---thus the mechanics of the system are described by the tilt angle θ, representing the angular displacement of the center of mass from directly above the feet, with the rod positioned normal to the ground. The pendulum is perfectly balanced when the mass is positioned directly above the pivot point; however, a very slight displacement of the mass from this position will cause the pendulum to tip over. Humans can overcome this difficulty and maintain standing balance, at which the system is at equilibrium, by maintaining active control of the position of their center of mass relative to the fulcrum formed by their feet. For small displacements, the ankles work to exert a torque that counteracts gravity and prevents the individual from falling over. The timescale that this restoring force must act to recover equilibrium is proportional to the period of a simple pendulum, demonstrated as T= 2Ď€ √L/g Precise measurements of active balancing can be made by filming an individual standing, and digitally tracking the location of the center of mass after the individual is tilted forward by a known angular displacement at t=0 . A sample figure showing the angular response is given in Figure 2. Surprisingly, these measurements show that the seemingly inert process of standing consists of many coordinated ankle motions that stabilize the body after it undergoes slight deflections.

25


MCAT Practice Test 1.pages

Figure 1: The analogy between a standing patient and an inverted pendulum.

26


MCAT Practice Test 1.pages

Figure 2: Simulated data showing the time-varying tilt angle of a standing individual who was tilted forward by 0.2 radians at t = 0, and who is actively returning to their original vertical standing position.

Concept adapted from: Winter, D. A. (1995). Human balance and posture control during standing and walking. Gait & Posture, 3(4), 193-214.

27


MCAT Practice Test 1.pages

26. Which of the following correctly describe the difference between an inverted pendulum (as shown in in Figure 1) and a standard pendulum with identical length and mass? I. An inverted pendulum has minimal kinetic energy when it reaches its equilibrium point II. An inverted pendulum requires a non-gravitational restoring force to remain in equilibrium III. An inverted pendulum reaches a higher maximum gravitational torque Please choose from one of the following options. 1 I and II 2 I, II, and III 3 I and III 4 II and III 27. Suppose a standing person loses their balance by tripping forward by a fixed angle (theta). Which of the following quantities would be greater for a taller person than a short person of equal mass? Please choose from one of the following options. 1 The torque that must be exerted by the ankles in order to restore balance. 2 The rate at which the ankles must exert a given torque in order to regain balance 3 The total gravitational force acting on the center of mass 4 The initial angular speed immediately after tripping forward

28


MCAT Practice Test 1.pages

28. In Figure 2, which of the following sets the initial amplitude at t = 0? Please choose from one of the following options. 1 The mass of the individual 2 The magnitude of gravitational acceleration 3 The angular distance by which the person initially tilts forward 4 The rate of the sudden tilt just before t=0

29. In Figure 2, which of the following determines the frequency of the decaying oscillations? Please choose from one of the following options. 1 The mass of the individual 2 The rate of the sudden tilt just before t=0. 3 The angular distance by which the person initially tilts forward 4 The magnitude of gravitational acceleration

30. Which of the following provides the most likely explanation for the decrease in amplitude of successive peaks of the sinusoidal response in Figure 2? Please choose from one of the following options. 1 Fatigue in the ankles providing the restoring force. 2 As the tilt angle changes, the torque decreases. 3 Gravitational potential energy being dissipated as heat. 4 Energy exiting the system due to damping effects.

29


MCAT Practice Test 1.pages

Lyophilisation, or freeze-drying, is a technique of dehydration which utilizes low pressure, low temperature environments to induce the sublimation of water content from a material. Sublimation is the process converting a substance from a solid directly to a gas, as shown in Figure 1. Figure 1. Generalized diagram indicating the phase change of sublimation.

30


MCAT Practice Test 1.pages

Perhaps the most recognizable form of freeze-drying is freeze-dried ice cream, sometimes also called ‘astronaut ice cream’ due to its development and use in the Apollo space missions; however, lyophilisation also has many applications in the pharmaceutical agricultural industry. Freeze-drying is ideal for preservation because low water content prevents enzymes and microorganisms from spoiling or degrading the substance. Additionally a freeze-dried substance can be rehydrated easily because of the microscopic pores left behind by the ice crystals when they sublimate away. This makes lyophilisation ideal for the long term storage and relatively fast reconstitution of substances, e.g. pharmaceuticals from an inert to an active form. There is a three stage process to lyophilisation. First the substance must be frozen, usually at very low temperatures between -50°C and -80°C. Once frozen, the substance is placed in a vacuum on plates, and a small amount of heat is added to help sublime the water directly from solid ice to water vapor. This is known as the primary drying stage, which removes 95% of a material’s water content. It can last up to several days, and requires rigorously maintained conditions due to the sensitivity of the sublimating structure. Finally, there is a secondary drying stage, which involves the temperature being raised again slightly and pressure being lowered further to sublime as much of the last 5% of moisture as possible. Researchers have found in their own astronaut ice cream making experiments that varying the temperature conditions can influence the rate of sublimation, as well as the consistency of the dehydrated ‘cake’ product. Table 1 shows how various temperatures affect the overall structure of freeze-dried ice cream during the primary drying stage by increasing temperature.

31


MCAT Practice Test 1.pages

Table 1. Results of lyophilisation of ice cream under varying temperature conditions.

Condition # 1$ 2$ 3$ 4$ 5$ 6$ 7$ 8$ 9$ 10$ 11$ 12$ 13$ 14$

$ $ $ $ $ $ $ $ $ $ $ $ $ $

$ $ $ $ $ $ $ $ $ $ $ $ $ $

Sublimation rate (mg/h) 142.8$$ 30.3$ $ 273.5$$ 312.8$$ 184$ $ 160$ $ 348$ $ 49.6$ $ 35.7$ $ 23.8$ $ 161.1$$ 225.3$$ 124.9$$ 298.2$$

$ $ $ $ $ $ $ $ $ $ $ $ $ $

Temperature: Variable and Stable (°C) -49 to -37$ -41 to -39$ -31 to -23$ -25 to -14$ -26 to -23$ -21 to -18$ -20 to -17$ -47$ $ -44$ $ -42$ $ -38$ $ -28$ $ -22$ $ -11$ $

$ $ $ $ $ $ $ $ $ $ $ $ $ $

Cake Structure $ $ $ $ $ $ $

Collapsed Solid Solid Loose Loose Solid Loose Collapsed Loose Solid Solid Solid Solid Solid

Citation: Tables Adapted from: Xiang, Jun, Hey, Jeffery M., Liedtke, Volker, D.Q. Wang, Investigation of freeze–drying sublimation rates using a freeze–drying microbalance technique. International Journal of Pharmaceutics Volume 279, Issues 1–2, 26 July 2004, Pages 95–105 Nail SL1, Jiang S, Chongprasert S, Knopp SA., Fundamentals of freezedrying. Pharm Biotechnol. 2002;14:281-360.

32


MCAT Practice Test 1.pages

31. Following the secondary drying stage, researchers found that the water vapor would consistently rehydrate the freeze-dried ice-cream, ruining their cake structure. Which of the following would best prevent the rehydration? Please choose from one of the following options. 1 Increasing the temperature at which the ice cream is frozen to form less ice crystals 2 Utilizing a very slow sublimation rate in order to allow the water vapor to dissipate 3 Increasing the pressure during the drying stages to slow down the rate of sublimation 4 Decreasing the pressure during the drying stages to promote rapid sublimation 32. If the researchers were looking to maximize the speed of making ice cream while maintaining a solid consistency of their product, which condition should they provide for freeze-drying? Please choose from one of the following options. 1 2 3 4

Condition 3 Condition 7 Condition 12 Condition 14

33. During the sublimation of water from a solid to a gas, how is the temperature of water being affected? Please choose from one of the following options. 1 Temperature is decreasing because substances release heat during phase changes 2 Temperature does not change during a phase change 3 Temperature is increasing because it requires heat to induce a phase change 4 The temperature varies during a phase change because it is dependent upon pressure

33


MCAT Practice Test 1.pages

34. Which of the following means of heat transfer would be least effective in warming a substance during the drying stages of lyophilisation? Please choose from one of the following options. 1 Conduction 2 Radiation 3 Convection 4 All three means of transfer would be equally useful

35. Often, scientists will place a super-cooled (−50 °C) condenser plate in the freeze-drying vacuum to prevent water vapor from rehydrating their freeze-dried product. How would a condenser plate help to prevent rehydration of the product? Please choose from one of the following options. 1 The condenser plate could condense water into a liquid and have it drained away 2 The condenser plate could absorb heat radiated during the secondary drying stage to prevent over-heating of the product 3 The condenser plate could act as a surface for water vapor to undergo deposition 4 The condenser plate could keep the environment cooler during the primary drying stage so that none of the initial ice crystals melt into the product

34


MCAT Practice Test 1.pages

Questions 36. 37. 38. 39. 40. are NOT based on passages. 36. Hypochlorous acid dissociates in water to create hydronium ions and hypochlorite ions HOCl + H2O H3O+ + OCl- Suppose that additional hypochlorite ions are added to the solution. Which of the following correctly describes the resultant effect on the concentration of HOCl? Please choose from one of the following options. 1 It increases 2 It depends on the umber of hydronium ions 3 It increases 4 It decreases

37. Carbon monoxide (CO) is a potent competitive inhibitor of hemoglobin, and it has an affinity for hemoglobin over 200 times greater than oxygen. What is the formal charge of the oxygen on the molecule? Please choose from one of the following options. 1 +1 2 0 3 -1 4 -2

35


MCAT Practice Test 1.pages

38. 500 mL of water and 500 mL of dichloromethane (d = 1.3 g/mL) are added to a mixture containing benzoic acid, cresol, methoxyethane, and N-methylethanamine. Four solutions are available for extraction of the mixture: HCN, HCl, NaOH, and LiHCO3. After the initial wash, the top layer from each extraction was retained and one of the preceding solutions was added. Which of the following statements most accurately describes this procedure? Please choose from one of the following options. 1 Cresol must be extracted first with a strong mineral acid such as HCl.. 2 Benzoic acid can be extracted and isolated with either LiHCO3 or NaOH. 3 After washing with LiHCO3 and then NaOH, only Nmethylethanamine and methoxyethane remain in the separatory funnel. 4 Methoxyethane will only be extracted upon protonation into a carbocation. 39. A new drug is developed which selectively cleaves covalent bonds between two sulfur atoms of non-adjacent amino acids in a polypeptide chain. Which level of protein structure in affected molecules would be most directly affected by the drug? Please choose from one of the following options. 1 Primary structure 2 Quaternary structure 3 Tertiary structure 4 Secondary structure 40. Chemical reactions can be classified according to free energy changes. A chemical reaction has a ΔG of -686/kcal/mol. Is this an endergonic or exergonic reaction? How would the addition of catalyst change the ΔG this reaction? Please choose from one of the following options. 1 Exergonic, the catalyst would not reduce ΔG. 2 Endergonic, the catalyst would reduce the ΔG. 3 Exergonic, the catalyst would reduce the ΔG. 4 Endergonic, the catalyst would increase the ΔG.

36


MCAT Practice Test 1.pages

As a pump, the heart continually is increasing and decreasing the pressure it exerts on blood. During diastole, or the relaxation phase, blood flows into the various chambers (called the atria and ventricles) of the heart, and during systole blood is pumped out. The total mechanical energy involved in this process can be calculated simply by multiplying the pressure of the blood by the volume of blood ejected from a given chamber (stroke volume). Stroke volume is distinct from the total volume of blood in a heart chamber, because not all blood is ejected from a chamber during a contraction. This amount of energy transferred by the heart during a contraction is often referred to as ‘stroke work’. Much like work done by an internal combustion engine, we can treat the heart as a thermodynamic system, and visually represent stroke work with the use of a pressurevolume diagram, as seen in Figure 1. Figure 1. Generalized Cardiac Pressure-Volume Loop Diagram

Let’s say that this diagram describes the relationship of volume and pressure specifically for the left ventricle of a heart. Position 1 of the graph would then mark the opening of the mitral valve, which allows blood to fill

37


MCAT Practice Test 1.pages

the chamber from the left atrium (diastole). Position 2 marks the closing of the mitral valve, and the beginning of systole. The chamber begins to contract, increasing blood pressure until it raises high enough to push open the aortic valve, leading out of the heart (Position 3). Once blood is ejected from the chamber (Position 4), pressure will decrease and the cycle is completed, which begins diastole again. There are various ways which we can influence how much work the heart does in a given stroke. One way we can increase stroke work is by increasing the volume of blood in the chamber during diastolic filling, known as preload. In other words, because the volume of preload is directly proportional to both stroke volume and blood pressure, an increase or decrease of preload will also increase or decrease stroke work respectively. 41. Which area of the graph would be equal to the amount of work done to the system after systole? Please choose from one of the following options. 1 The area contained within the 4 points of the cycle 2 The area underneath the curve from 2 - 3 3

The area underneath line A from points 1

-2

4

The area underneath line C entirely from points 3

-4

42. From step 4 to step 1, no work is being done either to or by the system. What type of thermodynamic process would this be? Please choose from one of the following options. 1 Adiabatic process 2 Isobaric process 3 Isothermal process 4 Isochoric process

38


MCAT Practice Test 1.pages

43. How would a decrease in preload affect the shape of the Cardiac Pressure-Volume Loop? Please choose from one of the following options. 1 It would increase the area within the cycle, and shift it to the right 2 It would increase the area within the cycle, and shift it to the left 3 It would decrease the area within the cycle, and shift it to the right 4 It would decrease the area within cycle, and shift it to the left 44. Aortic stenosis is a condition which stiffens the aortic valve. This results in the need for more pressure to cause ejection during systole. Which of the following would represent a cardiac cycle of someone with aortic stenosis? Please choose from one of the following options.

1

39


MCAT Practice Test 1.pages

2

3

40


MCAT Practice Test 1.pages

4 45. An adiabatic process is a thermodynamic process that involves maintaining a constant entropy, that is, no heat is lost from the system during the process. Would the cardiac cycle be considered an adiabatic process? Please choose from one of the following options. 1 Yes, only mechanical energy from the heart is transferred directly to the blood 2 Yes, mechanical energy and elastic potential energy are transferred without an increase in entropy 3 No, some energy is lost as heat due to the chemical processes of cardiac muscle contraction 4 No, some energy is lost as heat due to the active process of opening heart valves

41


MCAT Practice Test 1.pages

A Carnot engine is a type of engine that allows for the maximum amount of thermodynamic energy to be transformed into work. Although it is physically impossible to create one, the Carnot model is useful for determining the maximum efficiency of an engine. Scientists interested in increasing the fuel efficiency of automobiles begin by developing a heat engine, the closest engineers have come to replicating a Carnot engine. The heat engine includes a piston can move up or down allowing work to be done on the gas or allow the gas to do work on the surroundings. Neon, a monatomic ideal gas, is placed in a sealed canister connected to the tightly fitted piston. The canister can be placed in a temperature controlled room which allows heat to be transferred to the gas or allows heat to flow out of the gas. At the start of each experimental trial the gas is reset to a temperature of 298K and atmospheric pressure (101,000Pa). Assume that + work refers to the case where the surroundings do work on the gas, and – work refers to the case where the gas does work on the surroundings. Figure 1. A diagram of the heat engine the scientists are using.

42


MCAT Practice Test 1.pages

46. During a certain trial, the canister is placed in a room which is at a temperature of 350K, and the piston is seen to move upwards. What can be said about the heat (Q) and the work done on the gas (W) during this trial? Please choose from one of the following options. 1 Q is -, and W is + 2 Q is -, and W is 3 Q is +, and W is 4 Q is +, and W is + 47. During a certain trial, the piston is prevented from moving and the canister is placed in a room which is at a temperature of 255K. During this trial what will happen to the average kinetic energy of the atoms of the gas in the canister and why? Please choose from one of the following options. 1 The average kinetic energy of the gas atoms will decrease since the heat will be negative 2 The average kinetic energy of the gas atoms will decrease since the work done by the gas will be positive 3 The average kinetic energy of the gas atoms will increase since the heat will be positive 4 The average kinetic energy of the gas atoms will decrease since the work done by the gas will be negative 48. During a certain trial a researcher does 3.2J of work by pushing the piston down. During the time while this work is done, the room in which the canister is placed absorbs 4.5J of heat from the gas. What can be concluded about the motion of the gas molecules after this trial compared to the motion before the trial? Please choose from one of the following options. 1 The average speeds of the gas molecules increase 2 The speeds of all gas molecules decrease 3 The average speeds of the gas molecules decrease 4 The speeds of the gas molecules does not change

43


MCAT Practice Test 1.pages

49. During a certain trial, the piston is prevented from moving and the canister is placed in a room, which is at a temperature of 450K. What can be concluded about the pressure of the gas? Please choose from one of the following options. 1 The pressure of the gas will decrease since the heat will be 2 The pressure of the gas will not change since no work can be done 3 The pressure of the gas will increase since the work done will be + 4 The pressure of the gas will increase since the heat will be + 50. During a certain trial, the pressure of the gas is tripled while the volume is cut in half. What can be said about the kinetic energy of the gas molecules? Please choose from one of the following options. 1 The average kinetic energy of the gas molecules decreases since the temperature of the gas decreases 2 The average kinetic energy of the gas molecules increases since the temperature of the gas increases 3 The average kinetic energy of the gas molecules does not change since temperature will not change 4 The average kinetic energy of the gas molecules increases since the temperature of the gas decreases

44


MCAT Practice Test 1.pages

Human skeletal muscle tissue is uniquely structured by specialized layers. Muscles are composed of a collection of muscle fibers. In turn, these fibers are made of closely packed myofibrils, or a subunit made of tubular muscle cells. These myofibrils are surrounded by sarcoplasm, a substance similar to cytoplasm in other cells. Finally, myofibrils are composed of repeated sections of sarcomeres, which contain actin and myosin filaments. Physiologists interested in testing muscle cell tension capacity and musculoskeletal dynamics removed a sarcomere from a primate and placed it in physiological laboratory conditions. They analyzed the sarcomere’s movements under a number of different scenarios, including movement. A simplified model of a section of this sarcomere is given in Figure 1. Figure 1. A section of the sarcomere used by the scientists.

The tension in the actin between each myosin section are summed up and referred to as a single tension. The tension following myosin section 1 (M1) is referred to as TA, and the tension following myosin section 2 (M2) is TB. This section of actin is directly connected to the sarcomere’s end.

45


MCAT Practice Test 1.pages

Scientists noted that M1 contained a abnormal mass, making its mass greater than the mass of M2. The mass of M1 = 60kDa, and the mass of M2 = 30kDa. 1.6x10^{-24} kg = 1kDa. Assume that frictional forces are negligible. 51. If the mass of M1 > the mass of M2, and the sarcomere is moving with constant velocity, what can be said about the tension in both actin sections? Please choose from one of the following options. 1 Both tensions are the same and are zero 2 Both tensions TA and TB are equal and are not equal to zero 3 TA is smaller than TB since actin B has to pull the mass of both boxes 4 TA is larger than TB since the mass of M1 is larger than the mass of M2 52. If the mass of M1 > the mass of M2 and the sarcomere is accelerating to the right, how will the tensions in the actin sections compare? Please choose from one of the following options. 1 Both tensions TA and TB are equal and are not equal to zero 2 TA is larger than TB since the mass of M1 is larger than the mass of M2 3 TA is smaller than TB since actin B has to pull the mass of both boxes 4 Both tensions are the same and are zero 53. During one trial, the acceleration is 2m/s^2 to the right. What calculation will give the tensions in the actin filaments during this trial? Please choose from one of the following options. 1 TA= 1.6x10^-24 x 60 x 2, TB= 1.6x10^-24 x (60+30) x 2 2

TA = 60 x 2, TB = 30 x 2

3

TA= 1.6x10^-24 x 60 x 2, TB= 1.6x10^-24

4

TA = 60 x 2, TB = (30+60) x 2

x 30 x 2

46


MCAT Practice Test 1.pages

54. If the maximum force an actin section can maintain before tearing is 9pN, what is the maximum acceleration the scientists could make the sarcomere go while maintaining its structural integrity? Please choose from one of the following options. 1 9x10^−12N = (60kDa x 1.6 x 10^-24k) x a 2 9x10^−9N = (60kDa x 1.6 x 10^-24k) x a 3 9x10^−12N = (30kDa x 1.6 x 10^-24k) x a 4 9x10^−9N = (30kDa x 1.6 x 10^-24k) x a 55. Assume tensions are distributed equally amongst actin filaments

in TA. What is the force F1 on one filament of the first actin section if acceleration is a? Please choose from one of the following options. 1 F1 = (60 x 1.6 x 10^-24)a x 4 2 F1 = 60a / 4 3 F1 = (60 x 1.6 x 10^-24)a / 4 4 F1 = (60 x 1.6 x 10^-24)a

47


MCAT Practice Test 1.pages

Questions 56. 57. 58. 59. are NOT based on passages. 56. Which of the following statements most accurately describes the comparison between the rate constant k and the equilibrium constant keq? Please choose from one of the following options. 1 A large k means that a reaction will go to equilibrium, and a large keq means that a reaction will go to completion quickly. 2 k is formed from the concentration of the reactants raised to their stoichiometric coefficients, while keq is formed from the concentration of the products over the reactants each raised to its stoichiometric coefficient. 3 Both constants are dependent on temperature, but only Keq is independent of concentration. 4 The rate constant k indicates how long the reaction will take to complete, and the equilibrium constant keq indicates the ratio of the product concentrations to the reactant concentrations at equilibrium.

48


MCAT Practice Test 1.pages

57. Which of the following statements can reasonably be deduced from the phase diagram of helium below?

Please choose from one of the following options. 1 Solid and gaseous helium never exist in equilibrium with each other at any temperature or pressure. 2 At atmospheric pressure, helium can exist in all three phases, as well as a supercritical fluid phase near absolute zero. 3 At room temperature, helium exists as a gas, and as pressure increases eventually becomes a solid. 4 Helium does not exist as a solid below 25 atmospheres due to its low mass. 49


MCAT Practice Test 1.pages

58. Which of the following statements best characterizes the data represented in the liquid-vapor phase diagram of carbon dioxide where the dotted line represents phase change?

Please choose from one of the following options. 1 Above the isotherm for 31degrees celcius, carbon dioxide exists only as a supercritical fluid. 2 Moving from point 1 to 2, carbon dioxide is undergoing condensation. 3 Moving left along the isotherm from point 1, the pressure increases drastically since carbon dioxide becomes an incompressible solid. 4 Point 5 is the critical point of carbon dioxide.

50


MCAT Practice Test 1.pages

59. Which of the following carboxylic acids has the highest boiling point? Please choose from one of the following options. 1 heptanoic acid 2 octanoic acid 3 nonanoic acid 4 decanoic acid

END OF PART I OF IV

51


MCAT Practice Test 1.pages

Part II of IV Critical Analysis and Reasoning Skills 53 Questions 90 Minutes

52


MCAT Practice Test 1.pages

The rationalizing of society can be conceptualized as the pursuit of efficiency, predictability, calculability, and control through technology. But rational systems inevitably spawn a series of irrationalities that result in the compromising and perhaps even the undermining of their rationality. Fast-food restaurants, which epitomize the rational model, proffer the fastest means of getting from a hungry state to a sated one, without surprises, at low cost, in a carnival-like setting suggesting that fun awaits the consumer at each visit. The wholesomeness of the food seems an insignificant consideration. Whereas in the past, working people were prepared to spend up to an hour preparing dinner, they now are impatient if a meal is not on the table within ten minutes. (For their part, some fast-food restaurants have developed chairs that become uncomfortable after about twenty minutes, to ensure that diners do not stay long.) Fast-food restaurants have preferentially recruited adolescent help, at least until recently, because this age group adjusts more easily than adults do to surrendering their autonomy to machines, rules, and procedures. Few skills are required on the job, so workers are asked to use only a minute portion of their abilities. This policy is irrational from the standpoint of the organization, since it could obtain much more from its employees for the money (however negligible) it pays them. These minimal skill demands are also irrational from the perspective of the employees, who are not allowed to think or to respond creatively to the demands of the work. These restrictions lead to high levels of resentment, job dissatisfaction, alienation, absenteeism, and turnover among workers in fast-food franchises. In fact, these businesses have the highest turnover rate of any industry in the U.S. The entire workforce of the fast-food industry turns over three times in a year. Although the simple, repetitive nature of the work makes it easy to replace those who leave, the organization would clearly benefit from keeping employees longer. The costs of hiring and training are magnified when the turnover rate is extraordinarily high. The application of the rational model to the house-building process in the 1950s and ’60s led to suburban communities consisting of nearly identical structures. Indeed, it was possible to wander into the residence of someone else and not to realize immediately that one was not at home. The more 53


MCAT Practice Test 1.pages

expensive developments were superficially more diversified, but their interior layouts assumed residents who were indistinguishable in their requirements. Furthermore, the planned communities themselves look very similar. Established trees are bulldozed to facilitate construction. In their place, a number of saplings, held up by posts and wire, are planted. Streets are laid out in symmetrical grid patterns. With such uniformity, suburbanites may well enter the wrong subdivision or become lost in their own. Many of Steven Spielberg’s films are set in such suburbs. Spielberg’s strategy is to lure the viewer into this highly repetitive world and then to have a completely unexpected event occur. For example, the film Poltergeist takes place in a conventional suburban household in which evil spirits ultimately disrupt the sameness. (The spirits first manifest themselves through another key element of the homogeneous society—the television set.) The great success of Spielberg’s films may be traceable to a longing for some unpredictability, even if it is bizarre and menacing, in increasingly routinized lives. Adapted from G. Ritzer, The McDonaldization of Society. ©1993 by Pine Forge Press. 1. The author’s argument suggests that the primary motive of employers who make humans work with machines is to: Please choose from one of the following options. 1 improve the quality of their products. 2 reduce the cost of wages and benefits. 3 avoid seeming to be behind the times. 4 increase the uniformity of procedures. 2. A common thread in the discussion of fast food and the discussion of suburban housing is that people today: Please choose from one of the following options. 1 are increasingly resistant to the regimentation of life. 2 expect their needs to be met at the lowest possible cost. 3 allow themselves to be treated as interchangeable. 4 are unable to discriminate among products that differ in quality. 54


MCAT Practice Test 1.pages

3. Information in the passage suggests that a rationalized travel agency would emphasize: Please choose from one of the following options. 1 planned tours to popular attractions with accommodations at large hotels. 2 computerized systems to provide low-cost customized itineraries. 3 personnel trained to make reservations but with little experience as travelers. 4 procedures that encourage problem-solving initiatives by managers. 4. Suppose that the employee responses to working conditions in fast-food franchises (paragraph 4) also apply to entry-level assembly line workers. In light of this information, the author’s main point in mentioning these responses is: Please choose from one of the following options. 1 weakened, since the fast-food industry is not unique in suppressing creativity. 2 weakened, since the monotony of work is not necessarily related to employee dissatisfaction. 3 strengthened, since predictability and employee turnover are associated in another context. 4 strengthened, since low wages and job dissatisfaction are associated in another context.

55


MCAT Practice Test 1.pages

Americans are a “positive” people. This is their reputation as well as their self-image. In the well-worn stereotype, they are upbeat, cheerful, and optimistic. Who would be churlish enough to challenge these happy features of the American personality? Take the business of positive “affect,” which refers to the mood they display to others through their smiles, their greetings, their professions of confidence and optimism. Scientists have found that the mere act of smiling can generate positive feelings within us, at least if the smile is not forced. In addition, recent studies show that happy feelings flit easily through social networks, so that one person’s good fortune can brighten the day even for only distantly connected others. Furthermore, psychologists agree that positive feelings can actually lengthen our lives and improve our health. People who report having positive feelings are more likely to participate in a rich social life, and social connectedness turns out to be an important defense against depression, which is a known risk factor for many physical illnesses. It is a sign of progress, then, that economists have begun to show an interest in using happiness rather than just the gross national product as a measure of an economy’s success. Happiness is, of course, a slippery thing to measure or define. Philosophers have debated what it is for centuries, and even if they were to define it simply as a greater frequency of positive feelings than negative ones, when they ask people if they are happy, they are asking them to arrive at some sort of average over many moods and moments. Surprisingly, when psychologists measure the relative happiness of nations, they routinely find that Americans are not, even in prosperous times and despite their vaunted positivity, very happy at all. A recent metaanalysis of over a hundred studies of self-reported happiness worldwide found Americans ranking only twenty-third. Americans account for twothirds of the global market for antidepressants, which happen also to be the most commonly prescribed drugs in the United States. How can Americans be so surpassingly “positive” in self-image and stereotype without being the world’s happiest and best-off people? The answer is that positivity is not so much their condition as it is part of their 56


MCAT Practice Test 1.pages

ideology—the way they explain the world and think they ought to function within it. That ideology is “positive thinking,” by which they usually mean two things. One is the generic content of positive thinking—that is, the positive thought itself—which can be summarized as “Things are pretty good right now, at least if you are willing to see silver linings, make lemonade out of lemons, etc., and things are going to get a whole lot better.” The second thing they mean by “positive thinking” is this practice of trying to think in a positive way. There is, they are told, a practical reason for undertaking this effort: positive thinking supposedly not only makes us feel optimistic but actually makes happy outcomes more likely. How can the mere process of thinking do this? In the rational explanation that many psychologists would offer today, optimism improves health, personal efficacy, confidence, and resilience, making it easier for us to accomplish our goals. A far less rational theory also runs rampant in American ideology —the idea that our thoughts can, in some mysterious way, directly affect the physical world. Negative thoughts somehow produce negative outcomes, while positive thoughts realize themselves in the form of health, prosperity, and success. For both rational and mystical reasons, then, the effort of positive thinking is said to be well worth our time and attention. Adapted from B. Ehrenreich, Bright-sided. ©2009 by Metropolitan Books. 5. According to the passage, positive feelings are: Please choose from one of the following options. 1 universal 2 hereditary 3 contagious 4 ephemeral

57


MCAT Practice Test 1.pages

6. Suppose that economists do start using happiness instead of the gross national product as a measure of an economy’s success. Information presented in the passage would predict which of the following? I. The transition will be fraught with difficulty. II. The gross national product of the United States will appear to decrease. III. The economy of the United States will be seen as relatively less successful than today's. Please choose from one of the following options. 1 I only 2 II only 3 I and III only 4 II and III only 7. What best represents the author’s explanation for why Americans can be “so surpassingly ‘positive’ in self-image and stereotype without being the world’s happiest and best-off people?” Please choose from one of the following options. 1 Americans’ positivity is not a true reflection of their affect. 2 Being “well-off” is not the same as being “happy.” 3 Stereotypes tend to be unwarranted generalizations. 4 Americans tend to have high rates of depression.

58


MCAT Practice Test 1.pages

For someone used to contemporary academic writing, reading the chapter on color in William Gladstone’s Studies on Homer and the Homeric Age (1858) comes as rather a shock—the shock of meeting an extraordinary mind. It is therefore all the more startling that Gladstone’s nineteenth century tour de force comes to such a strange conclusion: Homer and his contemporaries perceived the world in something closer to black and white than to full Technicolor. No one would deny that there is a wide gulf between Homer’s world and ours: in the millennia that separate us, empires have risen and fallen, religions and ideologies have come and gone, and science and technology have transformed our intellectual horizons and almost every aspect of daily life beyond all recognition. Surely one aspect that must have remained exactly the same since Homer’s day, even since time immemorial, would be the rich colors of nature: the blue of sky and sea, the glowing red of dawn, the green of fresh leaves. Gladstone says things are not the same, for many reasons. One, Homer uses the same word to denote colors which, according to us, are essentially different. For example, he describes as “violet” the sea, sheep, and iron. Two, Homer’s similes are so rich with sensible imagery, we expect to find color a frequent and prominent ingredient, and yet his poppies have never so much as a hint of scarlet. Three, Gladstone notes, Homer uses “black” about 170 times, “white” 100 times, “red” thirteen, “yellow” ten, “violet” six times, and the other colors even less often. Four, Homer’s color vocabulary is astonishingly small. There doesn’t seem to be anything equivalent to our orange or pink in Homer’s color palette; most striking is the lack of any word that could be taken to mean “blue.” What is more, Gladstone proves that the oddities in Homer’s Iliad and Odyssey could not have stemmed from any problems peculiar to Homer. “Violet-colored hair” was used by Pindar in his poems. Gladstone is well aware of the utter weirdness of his thesis—nothing less than universal color blindness among the ancient Greeks—so he tries to make it more palatable by evoking an evolutionary explanation for how sensitivity to colors could have increased over the generations. The perception of color, he says, seems natural to us only because humankind as a whole has undergone a progressive “education of the eye” over the 59


MCAT Practice Test 1.pages

last millennia. The eye’s ability to perceive and appreciate differences in color, he suggests, can improve with practice, and these acquired improvements are then passed on to offspring. But why, one may well ask, should this progressive refinement of color vision not have started much earlier than the Homeric period? Gladstone’s theory is that the appreciation of color as a property independent of a particular material develops only with the capacity to manipulate colors artificially. And that capacity, he notes, barely existed in Homer’s day: the art of dyeing was in its infancy, cultivation of flowers was not practiced, and almost all of the brightly colored objects we take for granted were entirely absent. Other than the ocean, people in Homer’s day may have gone through life without ever setting their eyes on a single blue object. Blue eyes, Gladstone explains, were in short supply; blue dyes, which are very difficult to manufacture, were practically unknown; and natural flowers that are truly blue are rare. Gladstone’s analysis was brilliant, but completely off course. Indeed, philologists, anthropologists, and even natural scientists would need decades to free themselves from the error of underestimating the power of culture. Adapted from G. Deutscher, Through the language glass: why the world looks different in other languages. ©2010 Metropolitan Books. 8. It can be inferred from the passage that the author believes which of the following about contemporary academic writing? Please choose from one of the following options. 1 Academic papers are typically not especially brilliant. 2 Academics seldom address color perception in their papers. 3 Academics often reach very strange conclusions in their papers. 4 Academic papers are usually outdated soon after they are written.

60


MCAT Practice Test 1.pages

9. It has been suggested that the Iliad and the Odyssey were a patchwork of a great number of popular ballads woven together from different poets, rather than a single work by a poet named Homer. If true, how would this affect the opinions expressed in the passage? Please choose from one of the following options. 1 It would strengthen Gladstone’s basic thesis. 2 It would weaken Gladstone’s basic thesis. 3 It would require a modification of Gladstone’s basic thesis. 4 It would not affect Gladstone’s basic thesis. 10. Gladstone would predict which of the following about the children of an interior decorator who easily distinguishes among scarlet, burgundy, and fuchsia? Please choose from one of the following options. 1 The children would be able to easily distinguish various versions of red. 2 The children would be drawn more to objects in various versions of red than to those of any other color. 3 The children would seldom bother mentioning what are to them obvious differences among various versions of red. 4 The children would need to practice distinguishing among various versions of red for years before achieving proficiency. 11. Homer’s sky is starry, or broad, or great, or iron, or violet; but it is never blue. How does this affect the opinions expressed in the passage? I. It supports Gladstone’s claim regarding Homer’s use of color. II. It extends Gladstone’s claim regarding Homer’s focus on nature. III. It challenges Gladstone’s claim regarding Homer’s penchant for strange imagery. Please choose from one of the following options. 1 I only 2 II only 3 I and III only 4 II and III only

61


MCAT Practice Test 1.pages

In the United Kingdom, Physical Education (PE) is compulsory in state schools until students reach the age of 16. That is, sports are compulsory for as long as formal education is mandated by law. Because there are many children who don’t want to participate in PE classes, I believe that students should be allowed a choice. If their parents agree, why should they be forced to jump on a trampoline or do calisthenics? PE class is different from other classes because it involves what one does with one’s body. We acknowledge the right of individuals to control their own bodies— to determine whether and when they have an operation, to determine where they go and what they do. Why is this any different? It is a red herring to say that PE makes any serious difference to people’s health. There are more effective ways of ensuring a healthy population than pushing children to run laps around a freezing sports field once a week. For example, schools could be addressing the poor diets young people have today and encouraging them to walk or bicycle to school rather than rely on the car. Furthermore, sports are a waste of school time and resources. One or two PE lessons a week make very little difference to an individual’s health, but they make a huge difference in a school’s budget. Mandatory PE requires a whole extra department in schools, wasting a great deal of money and time that could be better spent on academics. It also requires schools to be surrounded by a large amount of land for playing fields, making it prohibitively expensive to build new schools in urban areas. Given the average current pupil–teacher ratios, the quality of teaching in PE classes is necessarily low, and the classes may even be dangerous to students who are not properly supervised. Our children are burdened enough in schools already, especially at the older end of the system, with multiple examinations. PE simply adds, needlessly, to this hectic schedule. Many people argue that playing team sports builds character, encourages students to work with others, teaches children how to win and lose with good grace, and builds strong school spirit through competition with other institutions. It is often, they say, the experience of playing on a team together which builds the strongest friendships at school, friendships which endure for years afterwards. Many say the same benefits derive from the common endurance of prison. 62


MCAT Practice Test 1.pages

Injuries sustained through school sport and the psychological trauma of being bullied for sporting ineptitude can mark people for years after they have left school. On that note, in an increasingly litigious age, a compulsory rather than voluntary sports program is a liability. More and more schools are avoiding team games such as rugby, soccer, hockey, and football due to the realistic fear of lawsuits. Teamwork can be better developed through music, drama, and community projects without the need to encourage an ultra-competitive ethos. As for the argument that without compulsory PE, many members of society wouldn’t find out that they had a talent for a sport or even that they enjoyed it, students can discover this aptitude outside of school, without also discovering the bullying and humiliation that comes with PE classes more than with other lessons. The aim of compulsory PE isn’t being fulfilled at present in any case, as “sick notes” are produced with alarming regularity by parents complicit in their children’s wish to avoid it. Greater efforts to enforce it will only result in more deceit, children missing school for the entire day, or, in the most extreme cases, children being withdrawn from state education. Adapted from A. Deane, “Physical Education, Compulsory,” Creative Commons. ©2011 Creative Commons. 12. What is the function of the statement in the first paragraph that “PE class is different from other classes”? Please choose from one of the following options. 1 It is part of an argument why PE classes should be required. 2 It is part of an argument that PE classes improve people’s health. 3 It explains why students should only be exempt from PE with parental permission. 4 It explains why students should have a choice about whether to take PE while not having a choice about taking other compulsory classes.

63


MCAT Practice Test 1.pages

13. Which of the following assumptions is made by the author in relation to the argument about students’ hectic schedules? Please choose from one of the following options. 1 PE tends not to have a final examination. 2 PE tends not to have a heavy homework burden. 3 Compulsory PE, if eliminated, would not be replaced by another compulsory course. 4 It is unfair to require students in the higher grades to prepare for multiple examinations. 14. Assume as true that students are more likely to obtain specialist coaching at sports clubs outside of school than in school. How would this information be relevant to the passage? Please choose from one of the following options. 1 It would restate an objection to compulsory physical education classes. 2 It would support a point about discovering sports aptitude made in rebuttal. 3 It would directly challenge one of the author’s claims. 4 It would contradict one of the author’s examples. 15. The author’s central theme for the whole passage is: Please choose from one of the following options. 1 opposing formal educational mandates. 2 describing the consequences of making PE compulsory. 3 presenting reasons for why PE should not be compulsory. 4 advocating that PE be abolished in UK state schools.

64


MCAT Practice Test 1.pages

Although business people deserve more respect for their honesty than they receive, a common complaint is that they take advantage of consumers through dishonest advertising. Instead of providing useful information for making rational choices, advertisements often appeal to consumers’ emotions to persuade them to buy products regardless of need. This complaint is true and obvious to all but the most naïve people. Advertisements are designed to convince consumers to favor one product over others, and presenting solely unbiased and unemotional information would seldom be the best way to accomplish this goal. Thoughtful people recognize that politicians advertise themselves and their policy recommendations in similarly biased and emotional ways. The question is not whether businesspeople or politicians have the strongest moral commitment to truthfulness in advertising. Both groups will deviate from honest practices when they expect that the benefits of doing so will exceed the costs. The important question is “Who can most easily mislead their customers with emotional statements, unrealistic promises, and biased information: businesspeople or politicians?” People are less likely to be swayed by dishonesty and emotion when responding to business ads than when responding to political ads for two reasons. First, businesspeople are attempting to persuade people who are usually spending their own money; politicians are trying to persuade people who are deciding how they want to spend other people’s money. The motivation to minimize mistakes by carefully considering claims about costs and benefits before a decision is made and by evaluating those claims in light of post-decision experience is greater when one is bearing all of the cost of the decision than when others are bearing most of the cost. The second reason why misleading claims are less effective in promoting commercial products than in promoting political products is because the choices that consumers of commercial products make have more decisive effects on outcomes than do the choices of consumers of political products. When people purchase a product in the marketplace, they get the product they choose, and they get it because they chose it. The probability that a voter’s choice will be decisive is increasingly small in state and federal elections, and seldom greater than a fraction of one percent in most local elections. Given such a low probability of any one person’s vote 65


MCAT Practice Test 1.pages

determining the outcome of the election, voters have little motivation to be concerned about the accuracy of the political claims being made. One might think that professors would be more honest than both businesspeople and politicians when promoting their products’ value (that is, in their teaching and research). Unlike politicians, professors try to sell their products to customers who can decisively accept or reject them without being directly affected by how many others make different choices. However, many undergraduate students are glaringly indifferent to what professors have to say, so professors have more latitude than businesspeople to benefit from exaggerated and duplicitous claims. Professors have to be more restrained when publishing than when teaching because other professors will evaluate the truth of their published claims. It is true that academic promotions may be earned and scholarly reputations enhanced by exposing the errors in published work. However, professors are often less concerned with the truthfulness of articles written by other professors than one might think. Professors anxious to get their own articles and books published are often less interested in whether the publications they cite are correct than in whether the publications are accepted as correct by academics with views similar to their own—the people most likely to decide whether their books and articles will be published and cited. Adapted from D. Lee, “Why Businessmen Are More Honest than Preachers, Politicians, and Professors.” ©2010 The Independent Review. 16. Which of the following assumptions is most central to the author’s argument? Please choose from one of the following options. 1 Most products are designed to appeal to naïve and emotional consumers. 2 Products are more likely to be purchased when they are advertised than when they are not. 3 If businesspeople manufactured only products that people need, there would be few products on the market. 4 If products were evaluated according to objective information about them, people would often not prefer one over the other. 66


MCAT Practice Test 1.pages

17. The author implies which of the following about businesspeople and politicians? Please choose from one of the following options. 1 Neither are very thoughtful people. 2 Neither have a strong moral commitment to truthfulness. 3 Both have biased views about their customers and constituents, respectively. 4 Both are more concerned about advertising themselves than their products, respectively. 18. Suppose a politician is re-elected despite lying about his voting record . The passage suggests which of the following explanations? Please choose from one of the following options. 1 The politician made many contradictory statements during his or her campaign. 2 Voter demographics for the second election were significantly different than for the first. 3 Voters did not compare the politician’s behavior while in office with statements made during his or her campaign. 4 There was no consensus among voters regarding the costs and benefits of a second term in office for that politician. 19. The author most likely mentions probability in his discussion of voting behavior as reasoning for which of the following (paragraph 4)? I. To explain low voter turnout in state and federal elections II. To explain the prevalence of politicians’ dishonesty III. To explain why voters do not carefully consider political claims Please choose from one of the following options. 1 II only 2 III only 3 I and II only 4 I and III only

67


MCAT Practice Test 1.pages

Imagine that someone offers you and some other anonymous person $100 to share. The rules are strict and known to both players. The two of you are in separate rooms and cannot exchange information. A coin toss decides which of you will propose how to share the money. If you are the proposer you can make a single offer of how to split the sum, and the other person— the responder—can say yes or no. If the responder’s answer is yes, the deal goes ahead. If the answer is no, neither of you gets anything. In both cases, the game is over and will not be repeated. What will you do? Instinctively, many people feel they should offer 50 percent, because such a division is “fair” and therefore likely to be accepted. More daring people, however, think they might get away with offering somewhat less than half of the sum. You may not be surprised to learn that two-thirds of the offers are between 40 and 50 percent. Only four in 100 people offer less than 20 percent. Proposing such a small amount is risky, because it might be rejected. More than half of all responders reject offers that are less than 20 percent. But why should anyone reject an offer as “too small”? The responder has just two choices: take what is offered or receive nothing. The only rational option for a selfish individual is to accept any offer. A selfish proposer who is sure that the responder is also selfish will therefore make the smallest possible offer and keep the rest. This game-theory analysis, which assumes that people are selfish and rational, tells you that the proposer should offer the smallest possible share and the responder should accept it. But this is not how most people play the game. The scenario just described, called the Ultimatum Game, was devised some twenty years ago. Experimenters subsequently studied the Ultimatum Game intensively in many places using diverse sums. The results proved remarkably robust. Behavior in the game did not appreciably depend on the players’ sex, age, schooling, or numeracy. Moreover, the amount of money involved had surprisingly little effect on results. Yet the range of players remained limited, because the studies primarily involved people in more developed countries and often university students. Recently, a cross-cultural study in fifteen small-scale societies showed that there were sizable differences in the way some people play the Ultimatum 68


MCAT Practice Test 1.pages

Game. Within the Machiguenga tribe (from the Amazon) the mean offer was considerably lower than in typical Western-type civilizations—26 percent instead of 45 percent. Conversely, many members of the Au tribe (from Papua New Guinea) offered more than 50 percent. Cultural traditions in gift giving, and the strong obligations that result from accepting a gift, play a major role among some tribes, such as the Au. Yet despite these cultural variations, the outcome was always far from what rational analysis would dictate for selfish players. Most people all over the world place a high value on fair outcomes. For a long time, theoretical economists postulated a being called Homo economicus—a rational individual relentlessly bent on maximizing a purely selfish reward. But the lesson from the Ultimatum Game and similar experiments is that real people are a crossbreed of H. economicus and H. emoticus, a complicated hybrid species that can be ruled as much by emotion as by cold logic and selfishness. An interesting challenge is to understand how Darwinian evolution would produce creatures instilled with emotions and behaviors that do not immediately seem geared toward reaping the greatest benefit for individuals or their genes. Adapted from K. Sigmund, E. Fehr, and M.A. Nowak, “The economics of fair play.” ©2002 by Scientific American, Inc. 20.Which of the following statements is NOT as strongly supported by the passage? Please choose from one of the following options. 1 The rules of the Ultimatum Game are strict. 2 The results of the Ultimatum Game tend to be consistent. 3 Responders reject offers that are less than 20 percent because they consider such offers unfair. 4 Studies of the Ultimatum Game show sizable differences in the way some people play.

69


MCAT Practice Test 1.pages

21. In some trials of the Ultimatum Game, the proposed split is determined by a computer. When responders are aware of this, they are willing to accept considerably lower offers. Based on the passage, compared to the standard game played without a computer, these responses are more: Please choose from one of the following options. 1 in keeping with what rational analysis would dictate. 2 out of keeping with what rational analysis would dictate, 3 in keeping with what one would expect from Homo emoticus. 4 in keeping with what researchers would expect of the responses from people of non-Western cultures. 22. Assume a “fair” offer is defined as 50 percent, and responders behave rationally. Based on the discussion in paragraph 5, it can be reasonably assumed that the author believes that the Machiguenga tribe had a: Please choose from one of the following options. 1 lower percentage of fair offers than both typical Western societies and rational selfish players. 2 higher percentage of fair offers than both typical Western societies, and rational selfish players. 3 lower percentage of fair offers than typical Western societies, and a higher percentage of fair offers than rational selfish players. 4 higher percentage of fair offers than typical Western societies, and a lower percentage of fair offers than rational selfish players. 23. In the discussion of the Ultimatum Game, what is the significance of the statement that “the range of players remained limited” (paragraph 4)? Please choose from one of the following options. 1 The limited sample did not allow the experimenters to generalize about all people. 2 Limiting the range of players allowed the experimenters to better control the outcome of the game. 3 Limitations on the game led to mistaken conclusions by experimenters at that time. 4 The limited sample allowed the experimenters to better control the range of variables in the experiment. 70


MCAT Practice Test 1.pages

The tools we use to think change the ways in which we think. The invention of written language brought about a radical shift in how we process, organize, store, and transmit representations of the world. Although writing remains our primary information technology, today when we think about the impact of technology on our habits of mind, we think primarily of the computer. My first encounters with how computers change the way we think came soon after I joined the faculty at the Massachusetts Institute of Technology at the end of the era of the slide rule and the beginning of the era of the personal computer. At a lunch for new faculty members, several senior professors in engineering complained that the transition from slide rules to calculators had affected their students’ ability to deal with issues of scale. When students used slide rules, they had to insert decimal points themselves. The professors insisted that doing that required students to maintain a mental sense of scale, whereas those who relied on calculators made frequent errors in orders of magnitude. Additionally, the students with calculators had lost their ability to do “back of the envelope” calculations, and with that, an intuitive feel for the material. That same semester, I taught a course in the history of psychology. There, I experienced the impact of computational objects on students’ ideas about their emotional lives. My class had read Freud’s essay on slips of the tongue, with its famous first example: The chair of a parliamentary session opens a meeting by declaring it closed. The students discussed how Freud interpreted such errors as revealing a person’s mixed emotions. A computer science major disagreed with Freud’s approach. The mind, she argued, is a computer. And in a computational dictionary—like we have in the human mind—closed and open are designated by the same symbol, separated by a sign for opposition. Closed equals minus open. To substitute closed for open does not require the notion of ambivalence or conflict. “When the chairman made that substitution,” she declared, “a bit was dropped; a minus sign was lost. There was a power surge. No problem.” The young woman turned a Freudian slip into an informationprocessing error. An explanation in terms of meaning had become an explanation in terms of mechanism.

71


MCAT Practice Test 1.pages

Today, starting in elementary school, students use e-mail, word processing, computer simulations, and virtual communities. In the process, they are absorbing more than the content of what appears on their screens. They are learning new ways to think about what it means to know and understand. There are a number of areas where I see information technology encouraging changes in thinking. There can be no simple way of cataloging whether any particular change is good or bad. That is contested terrain. At every step we have to ask, as educators and citizens, whether current technology is leading us in directions that serve our human purposes. Such questions are not technical; they are social, moral, and political. For me, addressing that subjective side of computation is one of the more significant challenges for the next decade of information technology in higher education. Technology does not determine change, but it encourages us to take certain directions. If we make those directions clear, we can more easily exert human choice. Adapted from S. Turkle, How computers change the way we think. Š2004 by The Chronicle of Higher Education. 24. Based on the passage, the author most likely believes that it is important to understand the influence computers have on people because such understanding will: Please choose from one of the following options. 1 enable people to make computers serve human purposes. 2 increase the importance of information technology in the next decade. 3 improve people’s ability to deal with issues of scale. 4 help prove that the human mind is a computational object.

72


MCAT Practice Test 1.pages

25. Which of the following passage assertions is presented as evidence that computers are affecting people’s conception of the mind? Please choose from one of the following options. 1 Engineering students using calculators frequently make mistakes regarding orders of magnitude. 2 Students who used calculators lost their ability to do “back of the envelope” calculations. 3 A computer science major interpreted a Freudian slip as an information-processing error. 4 Addressing the subjective side of computation is a significant challenge for the next decade of information technology in higher education. 26. Of the following scenarios, which represents an example most similar to what the author probably means by the opening statement, “The tools we use to think change the ways in which we think”? Please choose from one of the following options. 1 After a power outage, a person creates a plan for coping with such events in the future. 2 An analysis of the sequences of clicks emitted by dolphins reveals structural similarity to aspects of human language. 3 A person gains a new appreciation for abstract painting after learning about a new theory of complementary colors. 4 An office manager increases productivity by installing new accounting software on the company’s computers.

73


MCAT Practice Test 1.pages

27. Which of the following passage assertions is most supported in the passage by evidence or examples? Please choose from one of the following options. 1 Computational objects have an impact on students’ ideas about their emotional lives. 2 When students used slide rules, they had to insert decimal points themselves. 3 Students who used calculators lost their ability to do “back of the envelope” calculations. 4 The invention of written language brought about a radical shift in how we process, organize, store, and transmit representations of the world.

74


MCAT Practice Test 1.pages

Deconstructionism, as applied to literary criticism, is a paradox about a paradox: It assumes that all discourse, even all historical narrative, is essentially disguised self-revelatory messages. Being subjective, the text has no fixed meaning, so when we read, we are prone to misread. Deconstructionism emerged from Paris and, notwithstanding its claim to universality, has an evident history. It is a manifestation of existential anxieties about presence and absence, reality and appearance. It developed via structuralism, with its emphasis on semantics and symbolism. From these sources it derived its fundamental premise: the endless slippage of the subject, the futility of any attempt to name reality. The premise suggests the disillusion attendant on the collapse of the two major forces in twentieth-century European thought: enlightened humanism and idealistic Marxism. Despite its origins, deconstructionism found its own best home in the United States, that historically dissociated construction of random meanings. (“America is deconstruction,” said its leading proponent, Jacques Derrida.) By the 1970s, deconstruction filled—perhaps better, emptied—the gap left in the humanities in the U.S. by the demise of the old “new criticism.” But what began as brilliant and creative analytic performances soon became classroom pedagogy. Throughout the decade, the seminar rooms on U.S. campuses—and then campuses worldwide—became workshops in deconstructionist practice. Junior misreaders worked away, becoming ever more like C.I.A. operatives, decoding false signals sent by a distant enemy, the writer. Deconstruction exalted itself with ever higher pretensions. As one academic critic exulted, “The history of literature is part of the history of criticism.” Deconstruction transformed everything into social commentary, easily making affinities with sexual and racial politics, two other militant philosophies that challenge the sanctity of text. It presented itself as a supra-ideological mode of analysis, exposing the ideological aberrations of others while seemingly possessing none itself. Any resistance that deconstructionists encountered was usually interpreted as censorious ignorance. As their approach prevailed, gangs of 75


MCAT Practice Test 1.pages

neodeconstructionists descended on the library with their critical services. One would demythologize, another decanonize, another dephallicize, another dehegemonize, another defame. Literature, as the deconstructionists frequently proved, had been written by entirely the wrong people for entirely the wrong reasons. Soon all that would be left of it would be a few bare bones of undecidable discourse and some tattered leather bindings. This frenzy would be called a conference of the Modern Language Association. The point that needs to be reaffirmed is that writing is an existential act, an imaginative exploration of ideas. It is, in fact, an expression of moral responsibility. Literature is not a subordinate category of social criticism. When writers are censored, imprisoned, killed, or threatened with death for their writings, it is not because their discourse is undecidable. If we are to take authors and their fate seriously, we must recognize that fiction is more than an opportunity for word games; we must honor it as a mode of radical discovery. We need an ambiance around writing that affirms its nature as creativity, as art, and that in a larger sense considers creativity a prime power in the making of intelligence, feeling, and morality. This was the position from which Jean-Paul Sartre with his freedom-affirming existentialism started the postwar debate of which deconstruction is a latter-day development. He started it because during the 1930s the word had been defamed and disfigured, the book burned, the writer erased, by forces that lay outside criticism, in history. Adapted from M. Bradbury, The scholar who misread history. Š1991 by New York Times. 28. Which of the following statements best expresses the passage author’s message? Please choose from one of the following options. 1 The true meaning of texts can never be decided. 2 Most literary criticism is now deconstructionism. 3 Deconstructionist and existentialist critics compete. 4 The most salient issue for literary critics should be creative merit.

76


MCAT Practice Test 1.pages

29. The passage implies that the declaration “America is deconstruction� (paragraph 2) means that: Please choose from one of the following options. 1 the constant revisions typical of U.S. culture capture the spirit of deconstructionism. 2 literature produced in the U.S. is especially suited to analysis by deconstruction. 3 U.S. literary critics developed the scholarly methods adopted by deconstructionism. 4 U.S. higher education is controlled by the political agenda of deconstructionism. 30. According to the views of the passage author, a responsible review of a stage production of Shakespeare's Julius Caesar would probably: Please choose from one of the following options. 1 point out evidence that the play promoted the interests of Shakespeare's gender and class. 2 argue that Shakespeare distorted history unintentionally because of his limited knowledge. 3 assume that Shakespeare's presentation of events reflected his artistic sensibility. 4 recommend the play as an entertaining introduction to Roman history. 31. What is the author’s primary purpose in discussing deconstructionism in paragraphs 3, 4, and 5 Please choose from one of the following options. 1 To denounce its triviality in order to strengthen the case for a contrasting vision 2 To acknowledge its existentialist principles in order to propose an extension of these principles 3 To reveal its lack of sympathy for writers as an argument for a more courteous manner by critics 4 To contrast its analytic method with an evaluative approach that may prove equally interesting

77


MCAT Practice Test 1.pages

Is there such a thing as free will? Perhaps the philosopher who has gotten closest to a sensible understanding of free will is Daniel C. Dennett, who thinks of the phenomenon as “the power to veto our urges and then to veto our vetoes . . . the power of imagination, to see and imagine futures.” Over the last few decades, science has made small but significant advances in understanding the relationship between conscious and unconscious thought, and the data are beginning to paint a picture that seems to validate Dennett’s views. In the 1970s, Benjamin Libet wired people to an electroencephalogram and measured when they reported having a particular conscious thought about an action and when the nerve impulses corresponding to the initiation of the actual action started. Astoundingly, the subjects had actually made (unconsciously) the decision to act measurably earlier than when they became aware of it consciously. The conscious awareness, in a sense, was a “story” that the higher cognitive parts of the brain told to account for the action. So science may be showing us that free will is more a feeling than a real manifestation of independent will. As psychologist Dan Wegner put it, “We see two tips of the iceberg, the thought and the action, and we draw a connection.” Libet’s position is a bit more moderate and is akin to Dennett’s. Libet says that free will is a form of veto power, filtering and sometimes blocking decisions provisionally made at the unconscious level. Quantum mechanics is sometimes brought into discussions of free will by supporters of pseudoscience because it is very technical and, more important, incomprehensible enough to lend that aura of scientific credibility without committing one to specific details. Some philosophers and scientists suggest that perhaps free will can be explained by occasional quantum fluctuations that, by interfering with subcellular phenomena in the brain, create a partial decoupling of our decision-making processes from the standard macroscopic laws of causality. This is nonsense, not only because we have absolutely no evidence of “quantum fluctuations” (whatever they are) at the brain level, but because, even if they did happen, they would—at most—generate random, not free, will. And random will is not one of those varieties of free will that is, in Dennett’s words, “worth having.” 78


MCAT Practice Test 1.pages

Another source of confusion between science and philosophy when it comes to free will is to be found in the rather vague concept of “emergent properties,” for example, the notion of free will being an emergent property of the higher brain’s functions. Even though some scientists are predisposed to reject emergent properties, “emergence” can actually be studied scientifically and is a rather common phenomenon. For example, when hydrogen and oxygen combine to form water, the resulting molecule has emergent physical–chemical properties, in the sense that the temperatures marking transition states are not simple functions of the properties of the individual atoms. Some philosophers have argued that emergence restricts the limits of reductionism, not because it isn’t “physics all the way down,” but because, frankly, a quantum mechanical description of, say, the Brooklyn Bridge isn’t going to be very helpful. Emergence entails that certain phenomena are best studied, and understood, at some levels of analysis rather than others, and free will may well fall into this category. To say that it is an emergent property of the brain is not a call for magic or pseudoscience, just the realization that neurobiology and psychology are better positioned than quantum mechanics to understand it. Adapted from M. Pigliucci, Can there be a science of free will? ©2007 the Committee for Skeptical Inquiry. 32. Based on the passage, which of the following elements of Dennett’s characterization of free will best corresponds to what Libet calls “decisions provisionally made at the unconscious level”? Please choose from one of the following options. 1 Urges 2 Vetoes 3 Vetoes of vetoes 4 Imagining futures

79


MCAT Practice Test 1.pages

33. With which of the following statements about “emergent properties” does the passage information suggest that the passage author would be most likely to agree? Please choose from one of the following options. 1 Psychology is well positioned to study emergent properties of the functioning of the brain. 2 Supporters of pseudoscience tend to favor detailed explanations of free will. 3 Free will is an emergent property that is decoupled from the standard macroscopic laws of causality. 4 Emergent properties of molecules are just as easy to study as the properties of their individual atoms. 34. Assume that the phenomenon of happiness resists being described through reductionism. Given this assumption, and based on the passage, which of the following statements is most likely to be true? Please choose from one of the following options. 1 A description of the phenomenon of happiness can only be given in terms of one level of analysis. 2 A description of the phenomenon of happiness can be given without referring to emergent properties. 3 A description of the phenomenon of happiness requires referring to technical aspects of quantum mechanics. 4 A description of the phenomenon of happiness is more helpful in terms of psychology than in terms of physics. 35. Suppose that a scientist were to demonstrate to the passage author that quantum fluctuations occur at the subcellular level of the brain. Based on the fourth paragraph, is the author’s opinion that the quantum mechanical description of free will is “nonsense” likely to change? Please choose from one of the following options. 1 Yes, because quantum fluctuations would have been clearly defined 2 Yes, because quantum fluctuations would be subject to standard laws of causality 3 No, because quantum fluctuations would still generate random will 4 No, because some scientists reject quantum mechanics 80


MCAT Practice Test 1.pages

In a recent study, psychologist Anne Maass investigated the effects of courthouse architecture on the psychological well-being and cognitive processes of potential users. Specifically, she compared two courthouses located in Padova, Italy: the old courthouse, located in a former convent originally built in 1345, and the new courthouse, built in 1991 and designed by Gino Valle, an internationally known architect. Although serving or having served the same purpose, the two buildings have completely different styles—one is an old building with a rather residential look, warm colors, large windows, and a large wooden door, the other a massive, gray, semi-circular building, with narrow windows, and an entrance enclosed between two huge walls. When study participants were asked to imagine themselves accompanying a friend to the courthouse, they reported greater discomfort and stress when anticipating a trial in the modern building. However, contrary to predictions, this was true only when they were already familiar with the two buildings. It is possible that photographs reduced the actual impact of the architectural design, although this would contradict prior research by architect Gavin Stamp showing that distortions due to photographic presentation have negligible effects on preference. Another possibility for participants’ greater discomfort when imagining going to the new courthouse is that those with prior experience may have been exposed to the building from multiple angles, whereas unfamiliar participants received information only about the building’s facade. It is important to note that participants did not generally dislike the new building. From the standpoint of general aesthetic distinctions such as beauty versus ugliness, no differences emerged between the two buildings; if anything, the new building was seen by the participants as slightly more attractive. The data suggest that participants responded more to the intimidating nature of the building than to its beauty. The most important result of Maass’s research is that courthouse architecture was found to affect the estimated likelihood of conviction. Participants were more pessimistic about the trial outcome when they imagined entering the new building than when they imagined entering the old one. (This occurred regardless of whether participants had any prior familiarity with the respective buildings.) It remains unclear exactly which 81


MCAT Practice Test 1.pages

architectural features are responsible for the observed shift in likelihood of conviction estimates. The modern building differs on so many dimensions (size, color, shape, building materials, age, and so on) from the old building that it is impossible to isolate their individual impact. Also, it may be the interaction of features that creates the overall impression of the building as intimidating. How exactly do architectural features affect social-cognitive processes such as likelihood estimates? One possibility is that design features affect the emotional well-being or mood of the user which, in turn, biases his or her thought processes. For example, the architectural characteristics of the new courthouse seem to have made hypothetical users feel anxious and tense, and a bad mood has been shown to induce negative thoughts and expectations. However, building type affected perceived likelihood of conviction also for those participants who showed no enhanced discomfort in reaction to the new building. Another and more plausible possibility is that the design features of the new courthouse activated specific thoughts and mental associations related to conviction. For example, some participants spontaneously commented that the new building has greater resemblance to a prison than to a courthouse; others mentioned that the two high walls enclosing the entrance give the impression that those who enter the building are already convicted. Adapted from A. Maass, “Intimidating Buildings: Can courthouse architecture affect perceived likelihood of conviction?” Environment and Behavior. ©2000 by Sage Publications, Inc. 36. According to Maass, people’s reactions to the two buildings were independent of: Please choose from one of the following options. 1 whether or not they considered the buildings to be attractive. 2 what they knew of the history of the buildings. 3 the relative size of the buildings. 4 the influence of their peers.

82


MCAT Practice Test 1.pages

37. In the passage, the author justifies rejecting the emotional-moodas-mediator explanation with which of these reasons: Please choose from one of the following options. 1 emotions do not influence our expectations nearly as much as we think they do. 2 participants without negative emotions were affected in the same way as those with negative emotions. 3 the study did not measure and compare the participants’ moods before and after their imagined entry into the courthouse. 4 the study did not compare the mood of those entering the new building with the mood of those who imagined entering the new building. 38. Which of the following examples is most consistent with Maass’s suggestion that the architectural design features can affect perceptions by activating “specific thoughts and mental associations” (final paragraph)? Please choose from one of the following options. 1 A popular bank has décor, colors, and accents suggestive of gold. 2 An unpopular car wash does not have a drive-through design. 3 A popular restaurant has a children’s play area outside. 4 An unpopular grocery store has bad produce.

83


MCAT Practice Test 1.pages

Today’s parents face a tough battle. Neighborhoods are a lot more complicated than they were in the 1960s: every culture, every religion, every idea, every different standard, lives right next door. Information is received at lightning speed via the Internet, and children can be caught up in this whirlwind, subjected to things that they are still too young to understand or are emotionally unfit to handle. Censorship seems to be an answer to the growing problem of how to care for and watch over our children. But books are meant for exploration, for questioning. Within a book’s pages, children are safe to explore their feelings and reflect on their own situations. Putting the right book into the hands of the right child has great value and changes lives. It can be empowering, motivating, and inspiring. Here in the United States, an ostensibly free country, one where people are encouraged and given the legal right to speak their minds, we have been balancing personal freedoms and rights. But our media challenge this balance every day. As consumers, we respect artists and allow them the freedom of expression. At the same time, we are aware that children are seeing some unsuitable situations—but we are not always in agreement about what we want our children to watch, hear, or read. One political solution is rating systems, intended to help parents pick appropriate material for their children based on content, theme, violence, language, nudity, sensuality, drug abuse, and other elements. However, the rating systems have not stopped today’s lyrics from becoming more explicit, our cable television system from containing more swearing and sexual content, and our movies from becoming bloodier and more violent. And despite all the warnings and all the ratings, children are still listening to these songs, watching these television shows, and renting these movies. The rating system may have convinced politicians, parents, and librarians that it could do the job of protecting their young. It may have given people a false sense of security. But in reality, it means nothing when no one is there to monitor children’s actions and discuss appropriate behavior. Parents have a vested interest in their children. Creating a home in which a child feels safe is their responsibility. Creating a home where a child can safely make mistakes is their responsibility. Home is the first place where a 84


MCAT Practice Test 1.pages

child learns right from wrong, good from bad, healthy from unhealthy. It is the parents’ job to give their child a good defense by helping them establish boundaries. School helps to reinforce these lessons. Teachers help children by challenging them, instructing them, and helping them move on to the next level of maturity and understanding. A teacher may know, before a parent, when a child is ready for the next level or is mature enough to handle a theme or topic. When there is communication and respect between parent and teacher, the child’s development is the winner. America is a free society and has plenty of forums where people can express their views: newspapers, radio, billboards, and the Internet. People can discuss their differences and learn from each other. Why shouldn’t we allow our children that same rich experience? Banning a book is about as helpful as using a match in a hurricane. It does not shed light on anything and gets blown around by a lot of wind. Nor does sticking a label on a problem make it go away. Only in discussing, in sharing comments and concerns, is there growth and understanding. Let us show our children that knowledge is the most empowering censor they can use. Adapted from L. Caravette, “Censorship: An unnecessary evil,” The Looking Glass : New Perspectives on Children's Literature. ©2008 New Perspectives on Children's Literature. 39. Which of the following explains the relationship between the claim "Children’s lives should be very structured, so that they cannot be exposed to any uncontrolled sources of information" and the claims in the opening paragraph of the passage? Please choose from one of the following options. 1 It provides an explanation for a difficulty implied in the opening paragraph. 2 It illustrates an example of the author’s preferred solution. 3 It both contradicts and resolves a difficulty implied in the opening paragraph. 4 It neither supports nor challenges claims made in the passage.

85


MCAT Practice Test 1.pages

40. The author’s attitude toward the censorship of children’s books is one of: Please choose from one of the following options. 1 cautious support. 2 staunch neutrality. 3 general disapproval. 4 confused ambivalence.

41. Which of the following is most like the rating systems described in paragraph 4? Please choose from one of the following options. 1 Teachers’ grades and comments that appear on student assignments. 2 Library catalogs that classify books according to various fiction and non-fiction genres. 3 Road signs that inform drivers of dangers and speed limits. 4 Nutritional information about calories and vitamin content appearing on product packages. 42. The author mentions both the legal right to speak our minds and our lack of agreement about what we want our children to watch, hear, and read most likely in order to: Please choose from one of the following options. 1 illustrate the need for censorship. 2 explain why librarians should monitor what children read. 3 characterize the culture of the United States relative to that of other countries. 4 explain why rating systems have been developed

86


MCAT Practice Test 1.pages

A throng of bearded men, in sad-colored garments, and gray, steeplecrowned hats, intermixed with women, was assembled in front of a wooden prison. The founders of a new colony, whatever Utopia of human virtue and happiness they might originally project, have invariably recognized it among their earliest practical necessities to allot a portion of the virgin soil as the site of a prison. Before this ugly edifice was a grass-plot, much overgrown with burdock, pig-weed, apple-peru, and such unsightly vegetation which evidently found something congenial in the soil that had so early borne the black flower of civilized society, a prison. But, on one side of the portal was a wild rose-bush, covered with its delicate gems, which might be imagined to offer their fragrance and fragile beauty to the prisoner as he went in, and to the condemned criminal as he came forth to his doom. This rose-bush, by a strange chance, has been kept alive in history. It may serve to symbolize some sweet moral blossom that may be found along the track, or relieve the darkening close of a tale of human frailty and sorrow. The door of the jail being opened from within, there appeared, like a black shadow emerging into sunshine, the grim and grisly presence of the townbeadle. He laid his right hand upon the shoulder of a young woman; until, on the threshold of the prison-door, she repelled him, by an action marked with natural dignity and force of character, and stepped into the open air, as if by her own free will. Hester Prynne bore in her arms an infant, who winked and turned aside its little face from the too vivid light of day. When the young woman- the mother of this child- stood fully revealed before the crowd, it seemed to be her first impulse to clasp the infant closely to her bosom; not so much by an impulse of motherly affection, as that she might thereby conceal a certain token, which was wrought or fastened into her dress. In moment, however, wisely judging that one token of her shame would but poorly serve to hide another, she took the baby on her arm, and, with a burning blush, and yet a haughty smile, and a glance that would not be abashed, looked around her townspeople and neighbors. On the breast of her gown, in fine red cloth, surrounded with an elaborate embroidery and fantastic flourishes of gold thread, appeared the letter A. It 87


MCAT Practice Test 1.pages

was so artistically done, and with so much fertility and gorgeous luxuriance of fancy; and which was of a splendor in accordance with the taste of the age, but greatly beyond what was allowed by the sumptuary regulations of the colony. The young woman was tall, with a figure of perfect elegance on a large scale. Those who had before known her and had expected to behold her dimmed and obscured by a disastrous cloud, were astonished to perceive how her beauty shone out, and made a halo of the misfortune and ignominy in which she was enveloped. But the point which drew all eyes, so that both men and women, who had been familiarly acquainted with Hester Prynne, were now impressed as if they beheld her for the first timewas that SCARLET LETTER, so fantastically embroidered and illuminated upon her bosom. It had the effect of a spell, taking her out of the ordinary relations with humanity and enclosing her in a sphere by herself. 43. Information in the passage tells the reader that which of the following are Hester Prynne’s two tokens of shame? Please choose from one of the following options. 1 The letter “A” on her chest and her baby. 2 Her public humiliation and her baby. 3 The letter “A” on her chest and her criminal record. 4 Her ostentatious dress and her baby. 44. Which of the following statements would the author most likely agree with? Please choose from one of the following options. 1 Hester’s rejection of the grim and grisly town beadle underscores the danger of anarchy in small communities. 2 The sad-colored garments worn in the community are meant to portray the destitution of early Puritan society. 3 The rose-bush among the unsightly vegetation in front of the prison represents the merciful tendencies of Puritan society. 4 The rosebush symbolizes that there is a natural moral sense that transcends the strict and less understanding moral order of the Puritans.

88


MCAT Practice Test 1.pages

45. All of the following examples of imagery are mentioned, but which one is NOT used by the author to support the theme of dignity? Please choose from one of the following options. 1 “halo of misfortune” 2 “she repelled him...and stepped into the open air” 3 “haughty smile” 4 “burning blush” 46. The author would most likely agree with which of the following statements about the prison system in modern-day society: Please choose from one of the following options. 1 Eliminating most crime from society is possible with a strong prison system in place. 2 Creating a perfect society completely free of crime is unrealistic. 3 Prisons are unnecessarily expensive and a waste of taxpayer dollars. 4 Incarceration is inhumane and should be forbidden by governments internationally. 47. Suppose there is a newcomer to the community in which Hester Prynne was imprisoned. The newcomer dresses lavishly, eats voraciously, and spends excessively. It is MOST LIKELY that the newcomer: Please choose from one of the following options. 1 would be embraced as a new member of the community. 2 would be swiftly tried, convicted, and executed. 3 would be someone who shares and respects Puritan values. 4 would be met with disapproval by the community.

89


MCAT Practice Test 1.pages

48. Which of the following assertions best captures an idea from the passage? Please choose from one of the following options. 1 Puritan society allowed only men to adorn bright colors during certain times of the year. 2 The townspeople were expecting Hester Prynne’s imprisonment to have more of a negative impact on her than it did. 3 Hester Prynne’s birth of a baby girl marked her formal incorporation into society as a valued mother. 4 The Scarlet Letter worn on Hester’ Prynne’s chest is an emblem worn by many of the women in the community.

90


MCAT Practice Test 1.pages

Writer Thomas Hardy fits neatly into the general scheme of the literature of the second half of the nineteenth century, for he was about midway between writers Matthew Arnold and Walt Whitman. He believed in the efficacy of the knowledge given to one by an understanding of science; but he felt that new points of view should be impregnated with the ancient lore of the past. He believed that old wine should be kept in new bottles. He refused to follow the writers who called themselves realists into the morass of words that they accumulated in their attempts to portray life and character as they thought they were. Likewise, he refused to give himself to pure impressionism. He endeavored to preserve a balance between objective reality and his interpretation of it. He was in the old sense of the word a literary artist. Hardy, of course, read the events of human life in the light of his conception of the Immanent Will. He believed that the skein of circumstance, woven blindly and flung forth indifferently, caught up in its web all human beings from the emperor in his palace to the unconscious lout lying drunk in the ditch. But Hardy was not satisfied to hold this view conjecturally. He scanned the pages of philosophy, of science, and of history to be certain that he read life aright. From them he evolved a view of life which has been called scientific determinism. It seemed to him that men moved as automata, each within his own sphere. Unseen forces played upon them; unseen powers directed them. These forces, the physical manifestation of the metaphysical Immanent Will, were three in number, to which all others were subordinated. They were the power of heredity, the shaping power of education, and the influence of environment. From them there was no escape, for every choice seemingly made by the individual, Hardy thought, was dictated by so many thousand unseen circumstances so interwoven that it was almost impossible to realize the extent to which one was enmeshed in them. For evidence to substantiate this conclusion he could point to the past and to the present. The Greeks, for example, believed that the three Fates directed every action, no matter how minute, of mortals, and Immortals- of the peasant plodding in the field, and of Zeus waving his machinations on the cloud-kissed brow of Olympus. The Christian era had introduced into the intellectual world the contrary idea of Free Will; but the world had split on that interpretation of life during the Reformation, when John Calvin gave 91


MCAT Practice Test 1.pages

to the world from the dark caverns of his mind the gloomy doctrine of Predestination. In the nineteenth century, the western world was probably equally divided between the theory of Free Will maintained by the Roman Catholic Church and a few protestant denominations, and the theory of Predestination held by all churches stemming from Calvinism. Unexpectedly, aid from an unsought source came to those who maintained that human actions were predetermined, for the evolutionary theory, expounded by Darwin and Huxley, and the psychology which grew from it, gave weight to the idea that Predeterminism fitted better with facts than the theory of Free Will. Once an anthropomorphic God was out of the picture and His place taken by an evolving Consciousness, or whatever the mind of man chose to substitute, it was almost necessary to believe in a theory of life similar to that held by Hardy. 49. Which of the following arguments would strengthen the author’s claim about the relationship between predestination and evolutionary theory? Please choose from one of the following options. 1 Evolutionary theory predicts change over time, not daily decisionmaking. 2 Calvinists believed in God and championed Predestination simultaneously. 3 Evolutionary theory is scientific, while Predestination often involves divine forces. 4 All human decisions are determined by the brain which is a byproduct of biological evolution. 50. According to the passage, which two viewpoints held contrary positions regarding the existence of Free-Will? Please choose from one of the following options. 1 Catholicism and Calvinism 2 Calvinism and Evolutionary Theory 3 Evolutionary Theory and Psychology 4 Ancient Greek philosophy and science

92


MCAT Practice Test 1.pages

51. Which of the following endeavors would BEST exemplify the sense of impressionism alluded to in paragraph 1? Please choose from one of the following options. 1 A sculpture of a young woman crying as she clutches the figure of a small baby in her arms. 2 A novel describing the day-to-day shopping habits of a middle-class housewife. 3 A poem detailing the dangerous and unsanitary working conditions of industrial society. 4 A painting of teenagers suspended in space to portray their social disconnection. 52. Based on the philosophical beliefs of Thomas Hardy described in the passage, what can be inferred about the beliefs of writers Matthew Arnold and Walt Whitman? Please choose from one of the following options. 1 One believed in hedonism while the other believed in asceticism. 2 One valued ancient tradition while the other valued modern progress. 3 One preferred new bottles and the other preferred old wine. 4 One believed in fate and the other believed in free will. 53. A central thesis in the passage is that Hardy: Please choose from one of the following options. 1 was persuaded by science that only heredity guided our choices. 2 embraced the cross-pollination of scientific, philosophical, and artistic views of human life. 3 favored Catholicism over Calvinism because it offered a more complete explanation of the divine. 4 preferred an anthropomorphic God over an evolving Consciousness.

END OF PART II OF IV

93


MCAT Practice Test 1.pages

Part III of IV Biological and Biochemical Foundations of Living Systems 59 Questions 95 Minutes

94


MCAT Practice Test 1.pages

Pedigree charts are used to analyze the inheritance of traits within a human population. If enough progeny data is available, careful analysis of the phenotypes can provide information about the genetic inheritance of a disease. Immunodeficiency disorders are caused when the immune system’s ability to fight infectious disease is compromised. While immunodeficiency disorders vary widely, they are typically characterized by the person’s inability to make sufficient amounts of immune cells or by the production of immune cells that are mutated such that they cannot function properly. Given the wide variety of disorders, pedigree charts can be useful in determining what form of Mendelian inheritance a particular disease follows. Bruton syndrome, is a rare genetic disorder that is characterized by a mutation in the Bruton’s tyrosine kinase gene, which blocks the ability of Bcells to develop. To better understand the inheritance patterns of Bruton’s syndrome, a family was followed for three generations (Figure 1). Figure 1. Pedigree of family with cases of Bruton’s syndrome

95


MCAT Practice Test 1.pages

Figure legend: Circles are female, squares are male, black-filled shapes indicate individuals with Bruton’s syndrome. Another study followed a family with a different immunodeficiency disorder called hyper-IgE syndrome (Figure 2). There are several forms of hyper-IgE syndrome, which is characterized by mutations that prevents proper neutrophil chemotaxis. This disease results in recurrent cold infections, eczema-like skin rashes, and lung infections. In this pedigree, the hyperIgE syndrome was characterized by a mutation in the STAT3 gene. Figure 2. Pedigree of family with cases of hyper-IgE syndrome

Figure legend: Circles are female, squares are male, black-filled shapes indicate individuals with hyper-IgE syndrome.

96


MCAT Practice Test 1.pages

1. Based on the pedigree analysis, what type of Mendelian inheritance does Bruton’s syndrome MOST LIKELY exhibit? Please choose from one of the following options. 1 Autosomal dominant 2 Autosomal recessive 3 X-linked dominant 4 X-linked recessive 2. Hyper-IgE syndrome shows an autosomal recessive pattern of inheritance. Based on the pedigree analysis in Figure 2, which member of the family is most likely NOT a carrier? Please choose from one of the following options. 1 Person number 8 2 Person number 5 3 Person number 3 4 Person number 1

3. In Figure 1, what is the genotype of the Bruton’s tyrosine kinase allele in person number 4 ? Please choose from one of the following options. 1 This person has two normal alleles 2 This person has two disease alleles 3 This person has one normal and one disease allele 4 This person only has one allele and it is normal 4. In Figure 2, what is the most likely genotype of the STAT3 gene in person number 3 ? Please choose from one of the following options. 1 This person has two normal alleles 2 This person only has one allele and it is normal 3 This person has one normal and one disease allele 4 This person has two disease alleles

97


MCAT Practice Test 1.pages

5. If a female who has Bruton’s syndrome married a male who did not have the disease, what phenotypes would their progeny exhibit? Please choose from one of the following options. 1 100% of the females will have Bruton’s syndrome, while 100% of the males will be normal 2 50% of the males and females will be normal and 50% will have Bruton’s syndrome. 3 There is not enough information to answer this questions. 4 100% of the females will be normal, while 100% of the males will have Bruton’s syndrome.

98


MCAT Practice Test 1.pages

Helicobacter pylori is a Gram negative, microaerophilic bacterium characterized by its ability to survive in highly acidic conditions. This infectious pathogen, most commonly associated with Peptic Ulcer Disease (PUD), is responsible for the development of 70% of gastric ulcers, and an estimated 80-95% of duodenal ulcers, which may lead to the development of certain cancers. Once the bacteria reach the stomach, they burrow through the protective mucosal layer and adhere to the gastric epithelium, causing local cell destruction and inflammation. Most commonly, H. pylori colonizes the antrum of the stomach—the distal end of the organ which is most closely associated with the opening to the small intestine, the pyloric sphincter. Figure 1: Anatomical distribution of various secretory cell types throughout the stomach.

99


MCAT Practice Test 1.pages

Due to the relatively localized nature of H. pylori infection, it is not surprising that on microscopic analysis of a stomach afflicted by PUD, a marked destruction in somatostatin-producing D-cells is noted. The decreased presence of D-cells relative to other cell types within the stomach is accompanied by measurable increases in gastric acid production, and a decrease in luminal pH. H. pylori is capable of surviving in these harsh conditions by two mechanisms. First, the mucous layer through which it burrows to reach the epithelial surface is characterized by a more neutral pH than the gastric lumen. Second, the organism is capable of producing the enzyme urease, the effect of which is to produce a more neutral environment in the area immediately surrounding the bacterium. A strong association between chronic H. pylori infection and the development of gastric cancer has been demonstrated, though few studies have been able to accurately assess whether this risk is more strongly associated with specific microscopic subtypes of cancer or their anatomical distribution. Retrospective studies have been of limited use in this regard, as H. pylori does not colonize existing areas of cancer or atrophy, and thus such a model cannot fully assess the etiology of a long established gastric cancer. A 2001 review of twelve prospective studies attempted to illustrate the association between gastric cancer risk and anatomical distribution (Cardia vs. non-Cardia) in patients with a known H. pylori infection. The results of this study are shown in Figure 2, which map the Odds Ratio (a measure of association between two variables, wherein a ratio not equal to “one� denotes some association) and 95% confidence interval for the relationship between cancer development and H. pylori for the given anatomical area. Figure 2: Association between chronic H.pylori infection and the development of gastric cancers compared across twelve prospective studies. Matched Odds Ratio (OR) and 95% confidence intervals for association between H.pylori infection and non-cardia cancer (Panel A) and cardia cancer (Panel B). The area of the black squares is proportional to the study size. The white diamond shows the OR value for all studies combined, with 95% confidence interval represented by horizontal spread.

100


MCAT Practice Test 1.pages

101


MCAT Practice Test 1.pages

Data adapted from: Webb, P. M., Law, M., Varhese, C., & Forman, D. Gastric cancer and Helicobacter pylori : a combined analysis of 12 case control studies nested within prospective cohorts. Gut, 49, 347-353. 6. What is the most likely sequelae in generating the decreased luminal pH following the destruction of antral D-cells in H. pylori infection? Please choose from one of the following options. 1 Decreased production of somatostatin→ decreased inhibition of chief cells→ increased gastric acid production by chief cells 2 Decreased production of somatostatin→ decreased inhibition of Dcells→ increased gastric acid production by D-cells 3 Decreased production of somatostatin→ decreased inhibition of parietal cells→ increased gastrin production by parietal cells 4 Decreased production of somatostatin→ decreased inhibition of Gcells→ increased gastrin production→ increased numbers of mature parietal cells 7. Under normal conditions, what stimulates the release of somatostatin from Antral D-cells? Please choose from one of the following options. 1 Low fat content in gastric chyme 2 Acetylcholine 3 Low gastric pH 4 Distention of the stomach 8. What is the function of urease? Please choose from one of the following options. 1 Catalyzes the formation of somatostatin from urea and CO2 2 Catalyzes the degradation of urea to ammonia and CO2 3 Catalyzes the formation of urea from ammonia and CO2 4 Catalyzes the degradation of somatostatin to urea and CO2

102


MCAT Practice Test 1.pages

9. What is the best conclusion to draw from the data shown in Figure 2? Please choose from one of the following options. 1 H. pylori is positively associated with the formation of non-cardia gastric cancers and negatively associated with the formation of cardia gastric cancers 2 H. pylori is positively correlated with the formation of non-cardia gastric cancers and is not correlated with the formation of cardia gastric cancers 3 H. pylori is positively associated with the formation of cardia gastric cancers and negatively associated with the formation of non-cardia gastric cancers 4 H. pylori is positively correlated with the formation of cardia gastric cancers and is not correlated with the formation of non-cardia gastric cancers 10. Which of the following characteristics does H. pylori likely demonstrate? Please choose from one of the following options. 1 Utilizes Oxygen 2 Teichoic acids 3 Single lipid bilayer 4 Thick peptidoglycan layer

103


MCAT Practice Test 1.pages

Collagen is the most abundant protein in mammals and forms the main component of connective tissue. Over 28 types of collagen have been identified and described in humans. However, type I collagen constitutes over 90% of collagen. Type I collagen is found in bone, tendon, ligament, and skin. This protein is composed of three polypeptide chains, which wind around one another into a triple-helix stabilized by covalent cross-linkages. Collagen is distinctive in that its subunits are composed of a regular arrangement of Gly-Pro-X, where X can be any amino acid. Each amino acid has a precise role, as the glycine side chain is the only one that is small enough to fit into the center of the triple helix. The angle of the proline residue enables the polypeptide chains to fold into the triple helical structure. Osteogenesis imperfecta, which affects 6 to 7 per 100,000 people worldwide, typically results from genetic mutations in the collagen genes. The most common form of osteogenesis imperfecta, type I, arises from a dominant mutation in the COL1A1 gene, which results in a deficiency in collagen production. Other types of osteogenesis imperfecta, including types II, III, and IV are caused by dominant mutations in the COL1A1 or COL1A2 genes that alter the structure of type I collagen. In all cases, people with osteogenesis imperfecta suffer from bones that fracture easily. 11. Where does the mutation that causes osteogenesis imperfecta first occur? Please choose from one of the following options. 1 Collagen gene 2 Collagen triple-helix 3 Collagen polypeptide 4 Collagen transcript

104


MCAT Practice Test 1.pages

12. If collagen were treated with Protease X, which cleaves at the carboxyl terminal of glycine residues, and Protease Y, which cleaves at the amino terminal of proline, then what would be the length of the majority of the resultant fragments? Please choose from one of the following options. 1 1 residue 2 2 residue 3 3 residue 4 4 residue 13. Purified collagen protein samples were collected from the tissue of four different subjects. Which cell isolate would correlate with i) a patient with osteogenesis imperfecta type I and ii) a patient with osteogenesis imperfecta type II?

Please choose from one of the following options. 1 i) Patient B ii) Patient C 2 i) Patient D ii) Patient A 3 i) Patient C ii) Patient B 4 i) Patient D ii) Patient B

105


MCAT Practice Test 1.pages

14. If a patient comes in with osteogenesis imperfecta type I, which of the following statements is most likely true? Please choose from one of the following options. 1 Both alleles of COL1A1 have the osteogenesis imperfecta type I mutation. 2 Both parents are recessive for the COL1A1 osteogenesis imperfecta type I mutation. 3 One of the parents has osteogenesis imperfecta type I. 4 One of the COL1A1 alleles has a silent mutation. 15. If one allele of COL1A1 was mutated, so that one motif was now Trp-Pro-X, where X can be any amino acid, what disease type would this patient most likely display? Please choose from one of the following options. 1 Osteogenesis imperfecta type IV 2 No disease state would be displayed 3 This mutation would be lethal 4 Osteogenesis imperfecta type I

106


MCAT Practice Test 1.pages

Questions 16. 17. 18. 19. 20. are NOT based on a passage. 16. Which of the following amino acids has a net negative charge at physiologic pH (~7.4)? Please choose from one of the following options. 1 Lysine 2 Glutamic Acid 3 Asparagine 4 Histidine 17. The lagging strand of a DNA molecule undergoing replication reads 3’-CGCATGTAGCGA-5’. What is the code of the DNA that is the template for the complementary leading strand of this segment? Please choose from one of the following options. 1 5’-GCGTACATCGCT-3’ 2 5’-CGCATGTAGCGA-3’ 3 3’-CGCATGTAGCGA-5’ 4 3’-GCGTACATCGCT-5’ 18. In sickle-cell disease, a glutamate -> valine substitution results in formation of HbS molecules, which: Please choose from one of the following options. 1 Aggregate abnormally and cannot adequately carry O2 2 Have abnormally high-affinity binding for O2 3 Stabilize the wall of the red blood cell against oxidative damage 4 Cause experience high levels of repulsion between neighboring HbS molecules

107


MCAT Practice Test 1.pages

19. ATP hydrolysis under standard biochemical conditions has ∆G < 0, implying which of the following about the reaction under these conditions? Please choose from one of the following options. 1 Cleavage of the phosphoanhydride bond that occurs during hydrolysis does not require the input of energy 2 The hydrolysis reaction decreases entropy 3 The hydrolysis reaction is exothermic 4 The hydrolysis reaction is spontaneous 20. Two hours after being administered a high dose of glucose patient A blood glucose concentration read 7.6 mmol/L, then after 6 hours his glucose dropped to 7.0 mmol/L. On the other hand patient B numbers were 8.3 mmol/L, and 8.1 mmol/L after 2 and 6 hours, respectively, for the same dose of glucose. What can be said about the hormonal changes and sugar metabolic pathway of each patient? Please choose from one of the following options. 1 Patient A experienced high secretion of insulin leading to glycogenesis 2 Patient B experienced high secretion of glucagon leading to glycolysis 3 Patient B experienced low secretion of glucagon leading to glycogenolysis 4 Patient A experienced low secretion of insulin leading to lipogenesis

108


MCAT Practice Test 1.pages

Dietary branched-chain amino acids (BCAA) are marketed to athletes as a supplement that stimulates protein formation in muscle and reduces muscle breakdown. Branched-chain amino acids are those amino acids that have a side chain consisting of an aliphatic group with a branch point, including leucine, isoleucine, and valine (Figure 1). They account for 35% of the essential amino acids found in muscle proteins. Figure 1. Branched-chain amino acid properties Amino Acid Valine$ $ Isoleucine$ $ Leucine$ $

R-group CH(CH3)2$ $ CH(CH3)CH2CH3 CH2CH(CH3)2

pKa amino group 9.74$$ 9.76$$ 9.74$$

pKA carboxyl group $

2.30 2.32 2.32

The medication rapamycin has important immunosuppressive effects and its mechanism has been well studied. The mammalian target of rapamycin (mTOR), is a conserved serine/threonine protein kinase that regulates protein metabolism. mTOR integrates intracellular inputs such as growth factors (in particular BCAA amino acids) to promote protein translation. Activated mTOR results in the phosphorylation of a translation initiation protein called eIF-4EBP1. Unphosphorylated eIF-4EBP1 blocks translation by sequestering a different translation initiation protein called eIF4E, which is responsible for binding the mRNA cap and bringing the mRNA transcript to the ribosome. Phosphorylation of eIF-4EBP1 by mTOR causes eIF-4EBP1 to release eIF4E, and results in the initiation of translation. Recently, studies in animal models have suggested that certain BCAAs are more effective than others in activating mTOR (Figure 2), suggesting that nutritional supplements could be more effectively targeted toward building muscle through this pathway.

109


MCAT Practice Test 1.pages

Figure 2. Percentage phosphorylated eIF-4EBP1

You may use a calculator for this question. 21. What is the pI, or isoelectric point, of I) isoleucine, II) leucine, and III) valine? Please choose from one of the following options. 1 I) 6.03 II)6.04 III)6.02 2 I) 6.04 II)6.03 III)6.02 3 I) 7.41 II)7.44 III)7.45 4 I) 7.44 II)7.41 III)7.45 22. Which of the following amino acid substitutions would likely be the least disruptive to protein functionality? Please choose from one of the following options. 1 leucine → arginine 2 isoleucine → lysine 3 valine → alanine 4 glutamic acid → glutamine

110


MCAT Practice Test 1.pages

23. In Figure 2, which mouse has a higher level of mTOR basal activity? Please choose from one of the following options. 1 Mouse 1 2 Mouse 2 3 Both mice have the same mTOR basal activity levels 4 Neither of the mice have any mTOR basal activity 24. In Figure 2, which BCAA increases the activity of mTOR? Please choose from one of the following options. 1 Valine 2 Isoleucine 3 Leucine 4 None of the BCAAs 25. If nutritional companies use the data presented in Figure 2 as a reason to take supplements, which of the following assumptions are they making? I. Any increase in eIF-4EBP1 phosphorylation is due to the mTOR pathway II. The BCAAs are only activating the mTOR pathway III. Increases in eIF-4EBP1 phosphorylation leads to increased protein levels Please choose from one of the following options. 1 I and II 2 II and III 3 I and III 4 I, II, and III

111


MCAT Practice Test 1.pages

Transition state analogues are molecules that closely resemble a subtrate’s transition state structure in a reaction catalyzed by enzymes. The study of transition state analogues is one of the few techniques of rational drug design used to develop new pharmaceuticals. One of the more well known drugs discovered through this technique is Oseltamivir, which is a potent therapy against the infectious influenza virus. Oseltamivir is a competitive inhibitor for the enzyme neuraminidase and prevents neuraminidase from acting on its intended substrate, sialic acid. Neuraminidase is normally found on the influenza virus protein coat and hydrolyzes sialic acid molecules found on the exterior of host cell membranes to allow newly made viral particles to exit the original host cell, and begin infecting other host cells. Through the tight binding of Oseltamivir to neuraminidase’s active site, this process is inhibited, and the infectivity of the influenza virus is reduced. Due to the fact that Oseltamivir is a transition state analogue of sialic acid, their chemical structures are quite similar, as shown below:

112


MCAT Practice Test 1.pages

26. Which reactant, aside from sialic acid, would be necessary for neuraminidase to catalyze its reaction? Please choose from one of the following options. 1 Oseltamivir 2 An electron source 3 A co-factor or co-enzyme 4 Water 27. The rate of the uncatalyzed hydrolysis of sialic acid is 107 times slower than the rate of the same reaction with the neuraminidase catalyst. What accounts for this difference? Please choose from one of the following options. 1 Neuraminidase increases the activation energy of the reaction, thereby making the reaction more kinetically favorable. 2 Neuraminidase increases the activation energy of the reaction, thereby making the reaction more thermodynamically favorable. 3 Neuraminidase decreases the activation energy of the reaction, thereby making the reaction more kinetically favorable. 4 Neuraminidase decreases the activation energy of the reaction, thereby making the reaction more thermodynamically favorable. 28. At what point during the course of the neuraminidase catalyzed reaction with sialic acid, does the substrate most closely resemble the Oseltamivir analogue? Please choose from one of the following options. 1 When the energy of the enzyme-substrate complex is at its highest point. 2 When the energy of the enzyme-substrate complex most closely resembles the energy level of the original reactants. 3 When the energy of the enzyme-substrate complex is at its lowest point. 4 When the energy of the enzyme-substrate complex most closely resembles the energy level of the resulting products.

113


MCAT Practice Test 1.pages

29. If a 5 mL sample of 0.1 mM neuraminidase is added to a flask containing equal parts Oseltamivir and sialic acid, which of the two substrates is more likely to be bound to the neuraminidase? Please choose from one of the following options. 1 Sialic acid, as it has a greater affinity for neuraminidase as it can more readily bind the enzyme’s transition state structure. 2 Oseltamivir, as it has a greater affinity for neuraminidase, as it can more readily bind the enzyme’s transition state structure. 3 Sialic acid, as it has a greater affinity for neuraminidase, as it can more readily bind the enzyme’s resting structure. 4 Oseltamivir, as it has a greater affinity for neuraminidase, as it can more readily bind the enzyme’s resting structure. 30. Which fundamental principle surrounding enzymes explains why transition state analogues are able to act as potent inhibitors of naturally occurring enzymes? Please choose from one of the following options. 1 Allosteric Inhibition 2 Enzyme Specificity 3 Proximity and Orientation Effects 4 The Induced Fit Theory

114


MCAT Practice Test 1.pages

An overriding trend in the medical profession is the movement toward increasingly patient-specific treatment protocols, or so called ‘personalized medicine’. In accordance with this transition away from generalized applications of medical knowledge, clinical and pharmacological research has taken a particular interest in evaluating the outcomes of patients with increasingly specific risk factors and co-morbidities. Figure 1: The renin-angiotensin-aldosterone system and the site of ACE Inhibitor action

Angiotensin-converting enzyme (ACE) inhibitors are a first line pharmacological therapy in the management of hypertension (high blood pressure) and congestive heart failure. The primary mechanism by which these drugs achieve lower blood pressures in hypertensive patients is due to the reduction in activity of ACE and thus the inhibition of the conversion of angiotensin I to angiotensin II. In addition, ACE Inhibitors have been shown to inhibit the degradation of the vasodilating peptide, bradykinin, by ACE. Further details of this mechanism are shown in Figure 1. Due to the multifactorial nature of the renin-angiotensin-aldosterone system (RAAS), several potential targets for pharmacological treatment are readily identified. Indeed, in addition to ACE Inhibitors, angiotensin receptor blockers (ARBs), as well as direct renin inhibitors are routinely used. 115


MCAT Practice Test 1.pages

Although all of these drugs are effective in the management of hypertension, each one does so by means of a different mechanism, oftentimes accompanied by unique secondary effects. The efficacies of each of these drugs are continually evaluated through clinical practice, and as new research and potential new applications come to light, treatment algorithms are regularly adjusted. A 1999 study, noting that a strong majority of hypertensive individuals also demonstrate insulin resistance and decreased peripheral glucose dispersal as well as hyperinsulinemia, examined the effects of ACE Inhibitors and ARBs on glucose transport in insulin resistant muscle. Obese rats were injected with water, Captopril (an ACE Inhibitor), bradykinin, or eprosartan (an ARB). After treatment, the rats were anesthetized, and both epitrochlearis muscles removed, and incubated in a solution of 8mM glucose, 32mM mannitol, and 0.1% bovine serum albumin (BSA). One muscle from each rat was incubated with the addition of a 2mMUnits/ mLinsulin medium. After twenty minutes of incubation, the muscles were dissolved and used to determine glucose transport activity. The results of the experiment are shown in Figure 2.

116


MCAT Practice Test 1.pages

Figure 2: Effects of acute treatment with Captopril, bradykinin, or Eprosartan on glucose transport activity. Controls (vehicle-treated) are demonstrated for each treatment group, and the glucose uptake in the absence (-) and presence (+) of insulin reported with the net increase above basal (∆) caused by insulin. A “*” denotes data that is statistically significant.

To further elucidate causal relationships and eliminate potential statistical confounding factors, the experiment is run again, this time including a treatment group which received a dose of “Drug X”. Drug X is a well documented agent that is known to produce significant anti-hypertensive effects in patients and experimental models. The mechanism by which this is achieved is through the direct inhibition of renin. Passage adapted from: Henriksen, E. J., Jacob, S., Kinnick, T. R., Youngblood, E. B., Schmit, M. B., & Dietze, G. J. ACE inhibition and glucose transport in insulin resistant muscle: roles of bradykinin and nitric oxide. American Journal of Physiology - Regulatory, Integrative and Comparative Physiology, 277.

117


MCAT Practice Test 1.pages

31. What is the effect of Aldosterone on blood pressure, where is it synthesized, and where is its major site of action? Please choose from one of the following options. 1 Increase; hypothalamus; distal nephron 2 Decrease; adrenal glands; loop of Henle 3 Decrease; hypothalamus; loop of Henle 4 Increase; adrenal glands; distal nephron 32. Based on this passage, which of the treatment groups from the initial experiment would the results of the new treatment group with “Drug X� most likely resemble and why? Please choose from one of the following options. 1 The Captopril group, because the actions of Drug X and Captopril both result in a net decrease in renin 2 The Eprosartan group, because the inhibition of renin would mimic the blood pressure effect of blocking AT receptors 3 The Eprosartan group, because neither Drug X nor Eprosartan would inhibit the degradation of bradykinin 4 The Captopril group, because the actions of Drug X and Captopril both occur higher in the RAAS pathway than Eprosartan 33. Which of the following activate the renin-angiotensin-aldosterone system? Please choose from one of the following options. 1 Increased blood volume 2 Decreased renal sympathetic activity 3 Increased early distal tubule osmolarity 4 Decreased blood pressure

118


MCAT Practice Test 1.pages

34. Based on the passage, what is the association between treatment with Captopril and insulin-resistant muscles? Please choose from one of the following options. 1 Increases in the response to insulin in insulin-resistant muscles 2 Decreases in the amount of available insulin in insulin-resistant muscles 3 Increases in the amount of available insulin in insulin-resistant muscles 4 Decreases in the response to insulin in insulin-resistant muscles 35. With respect to stroke volume and cardiac output, what physiologic changes in the heart would be expected after treatment with Captopril? Please choose from one of the following options. 1 Decreased cardiac output, increased stroke volume 2 Increased cardiac output, decreased stroke volume 3 Increased cardiac output, increased stroke volume 4 Decreased cardiac output, decreased stroke volume

119


MCAT Practice Test 1.pages

Question 36. 37. 38. 39. 40. are NOT based on a passage. 36. Which intermolecular process primarily drives the formation of a bilayer when phospholipids are added to water? Please choose from one of the following options. 1 Phospholipids self-assemble into a bilayer due to the strong affinity they have for each other. 2 A bilayer arrangement maximizes the strength of Van der Waals forces among phospholipids. 3 Lipids cause water to arrange in an ordered, unfavorable cage-like structure. Forcing lipids into a bilayer reduces this effect. 4 The ordered arrangement of a bilayer is more favorable than the disordered state of individual free-floating phospholipids. 37. Which of the following transcriptional control features are found in both prokaryotes and eukaryotes? Please choose from one of the following options. 1 Introns 2 Promoter repressors 3 5’ methyl-G capping 4 Differential splicing 38. A scientist is investigating a specimen in a laboratory. She is attempting to determine whether it is a virus or not. Which of the following would allow her to conclude that it is not a virus? Please choose from one of the following options. 1 The specimen is extremely small 2 The specimen has a protein coat 3 The specimen contains DNA and RNA 4 The specimen has no organelles

120


MCAT Practice Test 1.pages

39. Oogenesis, the process that creates female gametes, is halted at prophase I until puberty. Which of the following describes the DNA content of a female’s gametes during her childhood? Please choose from one of the following options. 1 46 chromosomes, 92 chromatids 2 23 chromosomes, 23 chromatids 3 46 chromosomes, 46 chromatids 4 23 chromosomes, 46 chromatids 40. A previously unknown cell type is discovered in the central nervous system of a laboratory strain of mice. In cell-culture experiments, researchers find that the cell type can self-renew, but can only differentiate into one other cell type. Which of the following best characterizes this cell type? Please choose from one of the following options. 1 Multipotent embryonic stem cell 2 Unipotent somatic stem cell 3 Multipotent somatic stem cell 4 Unipotent embryonic stem cell

121


MCAT Practice Test 1.pages

The existence of leptin, a 167 amino acid polypeptide hormone that functions to control appetite and feeding behavior, was hypothesized by D.L. Coleman in 1978, almost 20 years before advances in molecular biology allowed for the isolation of the hormone and the identification of its associated gene. Coleman derived his hypothesis from the interpretation of experiments he performed with mice harboring a genetic mutation called db. Mice with this mutation, called db/db mice, are obese, diabetic, and hyperphagic (exhibit uncontrolled eating). When Coleman connected the circulatory system of a lean mouse with the circulatory system of a db/db mouse (referred to as “parabiosis�), the lean mouse eventually died of starvation, while the db/db mouse lived on, unaffected. In order to explain this result, Coleman hypothesized that the db/db mice produce a circulating molecule that, when introduced to the lean mouse via parabiosis, causes the lean mouse to cease eating, eventually leading to death by starvation. Subsequent research confirmed Coleman’s hypothesis: db/db mice produce abnormally large quantities of leptin, and the introduction of these large quantities of hormone to the circulatory system of lean mice precipitates their starvation and eventual death. In addition to his studies with the db mutation, Coleman also studied mice with a different genetic mutation, occurring on a different chromosome, called ob. Mice with this mutation, called ob/ob mice, are obese, diabetic, and hyperphagic, just like db/db mice. The following table contrasts the results of parabiosis experiments involving db/db mice, lean mice, and ob/ ob mice.

122


MCAT Practice Test 1.pages

Table 1. Results of three parabiosis experiments: db/db with lean, db/db with ob/ob, and lean with ob/ob.

Experim ent one

db/db: no change observed

Lean: hypophagia, hypoinsulinemia, and hypoglycemia. Death by starvation.

Experim ent two

db/db: no change observed

ob/ob: hypophagia, hypoinsulinemia, hypoglycemia, and reduction in adipose tissue mass. Death by starvation

Experim ent three

Lean: no change observed

ob/ob: normalization of eating behavior, blood glucose levels, and circulating insulin levels. Reduction in adipose tissue mass.

41. How many nucleotides long is the gene that codes for the hormone leptin? Please choose from one of the following options. 1 Exactly (167 x 3) base pairs 2 Greater than (167 x 3) + 3 base pairs 3 Exactly (167 x 3) + 6 base pairs 4 Exactly (167 x 3) + 3 base pairs

123


MCAT Practice Test 1.pages

42. Which of the following statements represents the best explanation for why db/db mice produce abnormally high levels of leptin? Please choose from one of the following options. 1 The db gene codes for a transcription factor bound by leptin in the nucleus of adipocytes, and this transcription factor is constitutively active in db/db mice, resulting in leptin resistance in leptin-sensitive cells and hypersecretion of leptin by leptin-producing cells. 2 The db gene codes for a transmembrane receptor on the surface of leptin sensitive cells, and this transmembrane receptor is constitutively active in db/db mice, resulting in leptin resistance in leptin-sensitive cells and hypersecretion of leptin by leptin-producing cells. 3 The db gene codes for a transmembrane receptor on the surface of leptin sensitive cells, and this transmembrane receptor is rendered inactive in db/db mice, resulting in leptin resistance in leptin-sensitive cells and hypersecretion of leptin by leptin-producing cells. 4 The db gene codes for a transcription factor bound by leptin in the nucleus of adipocytes, and this transcription factor is inactivated in db/db mice, resulting in leptin resistance in leptin-sensitive cells and hypersecretion of leptin by leptin-producing cells. 43. A hypothetical mouse mutation is identified that results in the absence of any circulating leptin in mice harboring the mutation: which of the following statements describe a cellular mechanism that could explain this lack of circulating leptin? Assume that the mutation is a loss of function mutation. I. The mutation occurs in the leptin gene. II. The mutation occurs in the gene that codes for a chaperone protein involved in post-translational modification of leptin. III. The mutation occurs in the gene that codes for a transcription factor that acts as a co-repressor of the leptin gene. Please choose from one of the following options. 1 III only 2 I and II only 3 I and III only 4 I, II, and III

124


MCAT Practice Test 1.pages

44. Which of the following compensatory changes should one expect to see as a result of the increased concentration of plasma glucose in hyperglycemic ob/ob mice? Please choose from one of the following options. 1 Increased secretion of glucagon by pancreatic ι-cells. 2 Increased secretion of insulin by pancreatic β-cells. 3 Increased secretion of growth hormone by somatotropic cells of the anterior pituitary. 4 Increased secretion of cortisol by cells in the zone fasiculata of the adrenal cortex. 45. An experiment is designed in which synthetic leptin is administered intravenously to db/db mice and ob/ob mice: which strain would most likely experience a cessation of the symptoms associated with their respective genetic mutations? Please choose from one of the following options. 1 Db/db mice only. 2 Ob/ob mice only. 3 Both strains. 4 Neither strain.

125


MCAT Practice Test 1.pages

Despite their hard, persistent nature and relatively static proportions, human bones are an extraordinarily dynamic tissue that is continuously resorbed, remodeled, and renewed. The maintenance of these processes requires precisely regulated homeostatic mechanisms in order to preserve adequate bone strength, structure, and density. Bone resorption is triggered by parathyroid hormone (PTH) under conditions of low calcium. PTH binds to receptors on osteoblasts which upregulate the expression of the signalling protein RANKL. RANKL binds to the receptor RANK on osteoclast precursors, activating the transcription factor Nf-kB, which signals them to differentiate and activate into functioning osteoclasts. OPG, produced in osteoblasts, competitively blocks RANKL, thereby preventing signaling. Because of this, net bone formation or resorption can be said to be a function of the ratio of OPG to RANKL. Osteoblasts synthesize and arrange bone matrix proteins, their chief role being to deposit collagen and other proteins to form new bone. These important interactions are shown in Figure 1.

126


MCAT Practice Test 1.pages

Figure 1: Key cells and signaling molecules in bone remodeling. Osteoblasts drive bone formation while osteoclasts drive bone resorption. Osteoclast differentiation and activation involves signals from PTH, receptor-ligand interaction with RANK and Nf-kB activation.

Several diseases are known in which some part of this process has been disturbed, and thus proceeds out of balance. Osteoporosis is a disease characterized by decreased bone density and a change in bone structure wherein the bones become increasingly porous. Osteoporosis affects over half of the population of the United States over 50 years old and is overwhelmingly more common among postmenopausal women, largely due to the marked decrease in estrogen levels. Estrogen plays an important role in regulating levels of RANKL and OPG. Autosomal recessive osteopetrosis (ARO) on the other hand, often referred to as “marble bone disease�, is characterized by excessively thick and dense, albeit brittle, bones. ARO is a congenital condition, affecting individuals from birth due to a number of genetic anomalies, typically 127


MCAT Practice Test 1.pages

resulting in death by the age of ten. One variant results from a mutation in the gene coding for RANKL; others from defects in OPG. A hypothetical study hopes to determine the epidemiology of osteoporosis and ARO in the United States, and attaches a survey to an annual political election ballots routinely over ten years, with a yearly sample size of nearly 10 million. The survey asks simple yes/no questions regarding if the individual (the voter) has ever had either of the two conditions. The results are shown in Table 1. Table 1: Epidemiology of bone remodeling disorders. The survey responses for three selected years of the ten year sampling period are shown. Percentages represent the frequency of the indicated response out of all potential respondents (all voters), reported to the nearly tenth of a percent. Blank surveys, illegible surveys, and responses indicating a response of “Unsure� account for the remaining percentage of potential respondents, and are not shown in the Table.

128


MCAT Practice Test 1.pages

46. Assuming each disease is due to a single defect in cells involved in bone turnover, what are the most likely defects in osteoporosis and osteopetrosis, respectively? Please choose from one of the following options. 1 Osteoporosis is due to the increased function of osteoblasts, osteopetrosis is due to the decreased function of osteoblasts 2 Both osteoporosis and osteopetrosis are due to the increased function of osteoblasts 3 Both osteoporosis and osteopetrosis are due to the decreased function of osteoclasts 4 Osteoporosis is due to the increased function of osteoclasts, osteopetrosis is due to the decreased function of osteoclasts 47. From what primary, embryological germ layer is bone derived? Please choose from one of the following options. 1 Mesoderm 2 Ectoderm 3 Neural crest 4 Endoderm 48. Are the results of the epidemiology study valid, why/why not, and what can be concluded about the incidence of ARO in the United States from this study? Please choose from one of the following options. 1 Yes, the results are valid. The sample size is sufficient, the study is blinded, and the extensive timeframe of the study minimizes the effect of bias. It can be concluded that the number of new cases of ARO each year did not significantly change across ten years. 2 No, the results are not valid. Although the sample size is large, it is affected by non-response bias. The only conclusion that can be drawn is that there were fewer cases of ARO in the United States in 2000 than in 1990. 3 Yes, the results are valid. The sample size is large, randomized, and likely to represent the study population as a whole. It can be concluded that there have been no new cases of ARO each year. 4 No, the results are not valid. Although the sample size is large, it is affected by considerable bias. No conclusions about osteopetrosis can be made. 129


MCAT Practice Test 1.pages

49. ARO is associated with mutations in RANKL and OPG—what structural or functional changes in RANKL and OPG would most likely be found in ARO? Please choose from one of the following options. 1 Decreased affinity of RANKL for its receptor and increased production of functional OPG would each be found in ARO 2 Decreased affinity of RANKL for its receptor and decreased production of functional OPG would each be found in ARO 3 Increased affinity of RANKL for its receptor and decreased production of functional OPG would each be found in ARO 4 Increased affinity of RANKL for its receptor and increased production of functional OPG would each be found in ARO 50. What would be a likely effect of increased PTH secretion? Please choose from one of the following options. 1 Increased osteoclast inhibition 2 Increased OPG expression 3 Increased serum phosphate 4 Increased free calcium

130


MCAT Practice Test 1.pages

For decades one of world’s deadliest infectious diseases has been AIDS, brought about by HIV. According to the World Health Organization, in 2012 there were an estimated 35 million people living with HIV, and about 1.6 million deaths due to AIDS. It is found on every inhabited continent, and while treatment has improved drastically, there is no known cure. HIV is a cytopathic retrovirus – it carries its genetic information in RNA enclosed in a protein envelope surrounded with a heavy layer of glycosylation. Reverse transcriptase transcribes the RNA into DNA, which is then inserted into host cells. The main target of the virus are CD4 T cells, also known as T helper cells. HIV enters cells by binding receptor proteins on the host’s outer membrane. HIV proteins gp120 and gp41 bind the T cell’s CD4 receptor and a coreceptor. Most often, this coreceptor is chemokine receptor 5 (CKR-5). This co-receptor is coded for by the CCR-5 gene. Both receptors are required for the virus to enter and infect the cell.

131


MCAT Practice Test 1.pages

Figure 1. Basic structure of HIV virus. RNA is contained within a capsid, surrounded by a lipid membrane. This envelope contains docking proteins such as gp120 and gp41. Image from wikimedia courtesy of the US National Institute of Health Individuals homozygous or heterozygous for a mutation known as CKR-5Δ32 have shown resistance to certain strains of the virus. However, this mutation is only found in a small portion of the population.

Figure 2. Nucleotide sequence of CKR-5 wildtype (WT) and CKR-5Δ32 The only case of being cured of HIV is a man known as the Berlin Patient. In 2006, about 10 years after discovering he had HIV, Timothy Ray Brown was diagnosed with acute leukemia while living in Berlin. As part of his cancer therapy, Mr. Brown received two stem cell transplants from a donor who was homozygous for the CKR-5Δ32 mutation. At this time, he also stopped taking his antiretroviral therapy (ART) treatment. Within a year of the transplants, HIV was undetectable and the patient’s white blood count

132


MCAT Practice Test 1.pages

had drastically increased (Figure 3). Mr. Brown remains HIV-free to this day.

Figure 3. Genotype of CCR5 variants. The Berlin Patient’s CCR5 expression before and after stem cell transplant (SCT). Data adapted from: Hutter, G. (2009). Long-Term Control Of HIV ByDelta32/Delta32 Stem-Cell Transplantation. New England Journal of Medicine, 692-698. Liu R, Paxton WA, Choe S, Ceradini D, Martin SR, Horuk R, et al. (1996). Homozygous defect in HIV-1 coreceptor accounts for resistance of some multiply-exposed individuals to HIV-1 infection. Cell, 86: 367-77. WHO Media Centre. (2014). HIV Fact Sheet World Health Organization

133


MCAT Practice Test 1.pages

51. What sort of mutation is CKR-5 Δ32? Please choose from one of the following options. 1 Deletion 2 Missense 3 Insertion 4 Nonsense 52. Which of the following features of HIV does NOT hinder the immune response? Please choose from one of the following options. 1 The viral RNA genome 2 The immune system target 3 The glycosylated outer envelope 4 The error-prone reverse transcriptase 53. How did the Berlin Patient’s cells transform after treatment? Please choose from one of the following options. 1 CD4+ , CCR5+/+ -> CD4+, CCR5-/2 CD4+ , CCR5+/- -> CD4+, CCR5-/3 CD4+ , CCR5+/+ -> CD4+, CCR5+/4 CD4+ , CCR5+/ - -> CD4+, CCR5+/+ 54. If you were to design a vaccine for HIV, which of the following would be the best target? Please choose from one of the following options. 1 Gp120, because its active site remains relatively constant among HIV mutations 2 The protein capsid, because it would destroy the structural integrity of the virus 3 CCR5, because without it the HIV cannot get into the host cell 4 Gp41, because without it the gp120 would not be attached to the virus

134


MCAT Practice Test 1.pages

55. How did the stem cell transplant affect the immune system of the Berlin Patient? Please choose from one of the following options. 1 The transplant gave the Berlin Patient enough HIV resistant white blood cells to fight both the cancer and the virus 2 The transplant caused the Berlin Patient to begin to make HIV resistant white blood cells 3 The transplant allowed the Berlin Patient to manufacture white blood cells, which had been halted by the HIV 4 The transplant replaced the HIV target cells with the resistant white blood cells from the donor

135


MCAT Practice Test 1.pages

Questions 56. 57. 58. 59. are NOT based on a passage. 56. Which of the following structures is a part of the rhombencephalon? Please choose from one of the following options. 1 Temporal lobe 2 Thalamus 3 Substantia nigra 4 Medulla 57. What is the correct path through the circulatory system which describes the passage of a blood clot originating in the left leg? Please choose from one of the following options. 1 Vena cava → right atrium → right ventricle → lungs → left atrium → left ventricle → aorta 2 Vena cava → left atrium → right atrium → lungs → left ventricle → right ventricle → aorta 3 Vena cava → left atrium → left ventricle → lungs → right atrium → right ventricle → aorta 4 Vena cava → right atrium → left atrium → lungs → right ventricle → left ventricle → aorta 58. People with albinism have a defect in tyrosinase, an oxidase that helps to control skin pigment production. In what epidermal layer is tyrosinase active? Please choose from one of the following options. 1 Stratum basale 2 Stratum spinosum 3 Stratum granulosum 4 Stratum lucidum

136


MCAT Practice Test 1.pages

59. In humans, which meiotic phase has the longest duration? Please choose from one of the following options. 1 Prophase I in females 2 Anaphase I in females 3 Metaphase II in females 4 Prophase II in males

End of Part III of IV

137


MCAT Practice Test 1.pages

Part IV of IV Psychological, Social and Biological Foundation of Behavior 59 Questions 95 Minutes

138


MCAT Practice Test 1.pages

A recent study found that obesity tends to spread like a “contagion” through a social network. In other words, when a person experiences weight gain, close friends in the same networks tend to gain weight as well. The investigators conducted a detailed analysis of a mass network of 12,067 people who had been closely followed over 32 years, from 1971 to 2003. In the study, 5,124people were used as key subjects, or “egos”, whose behavior was analyzed. Any persons linked to the egos serve as “alters”— those who may influence ego behavior. The researchers examined several aspects of obesity spread, such as clustering of obese persons within the network, association of weight gain among an individual’s social contacts, degree of dependence of association of social ties, and influence of gender or geographical distance. The researchers found that there were discernible clusters of obese persons (BMI ≥ 30) in the network at all time points. Figure 1 shows some results from the study. The extent of interpersonal association in obesity was evaluated with regression analysis. Homophily was taken into account by including a measurement of the alter’s obesity. The researchers evaluated the possible role of unobserved contemporaneous events by separately analyzing models of subgroups of the data involving ego-alter pairings. In particular, three types of “directional” friendships are defined: 1) an “ego-perceived friend” in which the ego identifies the alter as a friend;2) an “alter-perceived friend” in which the alter identifies the ego as a friend; 3) a “mutual friend” in which the identification is reciprocal. Familial ties (parents, siblings) and marital ties (spouses) are treated as reciprocal. “Immediate neighbor” denotes the geographical distance between an alter and an ego. For example, from the results, we can see that if an ego stated that an alter was a friend, the ego’s chances of becoming obese appears to increase by 57% (“risk of obesity”).

139


MCAT Practice Test 1.pages

Figure 1: Percentage increase in obesity risk for an ego based upon his/her relationship with an alter. The dependent variable in each model is the obesity of the ego. Independent variables include a timelagged measurement of the ego’s obesity, the obesity of the alter, a time-lagged measurement of the alter’s obesity, the ego’s sex, age, and education. Mean effect sizes (solid black dot) and 95% confidence intervals (line) are shown.

In later studies on the relationship between social networks and health behaviors, one of the researchers further found that existing social ties (especially close friendships) are more likely to dissolve between people who have health traits that are dissimilar, including health traits that are immutable such as height and personality, and traits that are mutable such as BMI, blood pressure, etc. In particular, those with similar BMIs are less likely to dissolve existing ties and more likely to form ties. Another study demonstrated that food choices also were made in accordance to social networks. In particular, spouses showed the strongest influence in food consumption behaviors, controlling for social contextual factors. Across all peers (spouses, siblings, friends), eating patterns that were most likely to be shared were “alcohol and snacks”.

140


MCAT Practice Test 1.pages

Source: Adapted from Christakis, N. A., & Fowler, J.H. (2007). The Spread of Obesity in a Large Social Network over 32 Years. The New England Journal of Medicine. 357(4), 370-379. O'Malley, J., & Christakis, N. A. (2011). Longitudinal analysis of large social networks: Estimating the effect of health traits on changes in friendship ties. Statistics in Medicine. M. A. 30(9), 950-964. Pachucki, M.A., Jacques, P.F., & Christakis, N.A. (2011). Social Network Concordance in Food Choice Among Spouses, Friends, and Siblings.American Journal of Public Health, 101(11), 2170-2177.

1. Which conclusion is best supported by the findings in Figure 1? Please choose from one of the following options. 1 Friends of opposite genders only marginally increased the likelihood of obesity for the ego. 2 Obese persons do not seem to selectively form social ties only with other obese persons. 3 There is almost no effect on the ego when someone in the same geographic proximity gained weight. 4 If a mutual friend living far away gained weight, the ego would not be more likely to gain weight. 2. Which of the following is NOT a plausible policy implication of the peer effects findings in the above passage? Please choose from one of the following options. 1 It may be possible to exploit variations in people’s social network position to target interventions where they may be more effective in generating benefits for the group, such as key nodes who exert stronger influence on others. 2 Group-level interventions such as Alcoholics Anonymous or other support groups that can serve as a set of artificial social network ties may be more successful than individual-level interventions. 3 Shared external sources contribute to obesity, so interventions that take common environmental factors into consideration can effectively target individual health behavior. 4 If we spend $500 to get a person to quit smoking, this person’s quitting may in turn result in his or her social contacts quitting, increasing the cost-effectiveness of an intervention.

141


MCAT Practice Test 1.pages

3. If the studies were combined to investigate the effects of social networks on food selection and how this food selection is associated with the likelihood of obesity, how would this change the design of the first study (social network effects on obesity)? Please choose from one of the following options. 1 A new dependent variable would be added. 2 A new mediating variable would be added. 3 A new independent variable would be added. 4 The variables would not change. 4. Which of the following statement best describes the concept of homophily? Please choose from one of the following options. 1 Smokers are more likely to experience cognitive dissonance with other smokers, compared to non-smokers. 2 Obese people are more likely to change their unhealthy behaviors around non-obese people. 3 Individuals with similar health risks such as high blood pressure are less likely to dissolve social ties with one another. 4 Individuals who are in the same geographical proximity are more likely to conform to the same health behaviors. 5. Which independent variable is most relevant for a study that solely investigates the impact of primary groups on health behaviors? Please choose from one of the following options. 1 Religious Affiliation 2 Sibling 3 Occupational Affiliation 4 Neighbor

142


MCAT Practice Test 1.pages

Significant dietary and nutritional differences can be found among racial and socioeconomic groups in the United States. These nutritional behavioral differences are the foci of many studies since they contribute to racial disparities in the incidence and prevalence of chronic disease and premature morbidity. Using survey data of over 60,000 participants collected by the US Department of Agriculture, Study 1 compared the dietary trends among 32,406 Black and White nonpregnant adults (18 years or older) of varying socioeconomic status (SES). The primary outcome was the score (0-16) on the Diet Quality Index (DQI), a composite of eight food-and-nutrient-based recommendations from the National Academy of Sciences, including areas such as “eating 5 or more servings daily of vegetables and fruits” and “limit total daily intake of sodium to 2,400 mg or less”. For each recommendation a person could score 0-2 according to specified intake amounts. A total score of 4 or less was considered to indicate a more healthy diet, and a value of 10 or more indicates a relatively less healthy diet. Categories of SES were based on education and income. Respondents with over 12 years of education and an income level over 350% of poverty level were categorized as “high SES”, and those with less than a highschool education and income less than 185% were classified as “low SES”. Three time periods were taken into account: 1965(I), 1977-1978(II), and 1989-1991( III ), to observe trends over time. Table 1 displays some results.

143


MCAT Practice Test 1.pages

More recently Study 2 adjusts for SES, and directly explores the association between race and nutrition. Using data from the 1993-1999 California Dietary Practices Survey, a researcher examines the differences between the nutritional behavior of Blacks and Whites (n=3,350). The researcher attempts to observe whether Blacks differ significantly from Whites in terms of health-related nutritional behaviors that have established associations to the development of heart disease, cancer, stroke, and diabetes. The results indicate that, even within the same SES group, Whites are more likely, on average, to exhibit healthy nutritional behaviors such as the consumption of at least five fruits, dairy products, high fiber cereals, lowfat dairy products, and avoiding the consumption of deep-fried foods and snacks (statistically significant). However, for categories such as “consuming wholegrain products” or “consuming beans” no significant differences were found among the two racial groups in the same SES groups.

144


MCAT Practice Test 1.pages

Sources: Adapted from Popkin, B.M., Siega-Riz, A.M. & Haines, P.S. . A Comparison of Dietary Trends among Racial and Socioeconomic Groups in the United States. New England Journal of Medicine,. Race and nutrition: an investigation of Black-White differences in health-related nutritional behaviors. Sociology of Health and Illness,

6. According to Table 1 which of the following is true? Please choose from one of the following options. 1 On average among Blacks, those in the low SES group had less healthy dietary behavior compared to those in the medium SES group across all time periods. 2 Differences in the mean DQI score according to race and SES have widened over time. 3 Dietary quality improved over time for both Blacks and Whites across all socioeconomic status. 4 Comparing Blacks and Whites in low and medium SES groups, the proportion of Blacks with poor diet quality exceeded that of Whites in each time period. 7. Consider the results from both Study 1 and Study 2. Assuming statistical significance for all findings presented, which of the following conclusion is best supported? Please choose from one of the following options. 1 In either study, no health-related racial disparities were found. 2 In Study 2 , the researcher finds that poor Whites have better diets than rich Whites. 3 In Study 2 the researcher shows that Blacks are more likely to have heart disease compared to Whites due to the lack of positive dietary behaviors. 4 In Study 1, results indicate that on average, poor Whites have better dietary behaviors than rich Whites.

145


MCAT Practice Test 1.pages

8. If another researcher finds that strong social support among Whites explains positive health behavior, which of the following is correct? Please choose from one of the following options. 1 Race is the independent variable, social support is the dependent variable. 2 Health behavior is the independent variable, race is the dependent variable. 3 Social support is the independent variable, race is the dependent variable. 4 Social support is the independent variable, health behavior is the dependent variable. 9. Socioeconomic gradients in health can be best explained by which of the following basic concept Please choose from one of the following options. 1 Intersectionality 2 Social exclusion 3 Social stratification 4 Class consciousness 10. To further understand how SES influences health behavior and outcomes, we must think about the processes in which health outcomes occur. Which of the following is NOT a causal mechanism through which SES may affect health outcomes? Please choose from one of the following options. 1 A person with higher SES has more resources to buffer health threats. 2 A person with lower SES is more likely to have mental health issues. 3 A person with lower SES is less likely to have social support or capital that would protect against mental health diseases. 4 A person with higher SES is less likely to be exposed to bad environmental conditions that contribute to diseases.

146


MCAT Practice Test 1.pages

Epidemiologically speaking, it is crucial to understand the contact structure through which disease travels. In studies of sexually transmitted diseases (STDs) in particular, the structure of sexual networks is critical for understanding STD diffusion. Generally, the network structures of disease diffusion and infection can be categorized into four types: the core infection model, the inverse core infection model, the bridging model, and the spanning tree model (see below). Figure 1. The network structure of four models of infection. For Panels A, B, and C, black circles denote actors that are high-activity or high-risk, and white circles indicate low-activity or low-risk actors. Panel D circles represent randomly selected individuals. Lines represent the relationship between two actors. The dotted lines in Panel A and B delineate the “core�.

Disease diffusion is widespread among adolescent populations. A study mapped the romantic and sexual relationships of an entire high school population of over 800 adolescents in a midsized town in the Midwestern US for a period of 18 months. Students were asked to identify their sexual (not dating) and romantic partners (dating) in the past 18 147


MCAT Practice Test 1.pages

months from a roster of other students attending their school. The study found that adolescent sexual networks are structured very differently from adult ones. In an adult sexual network, there is normally a core group of very sexually active people that links out to others. This can be envisioned as a transportation hub system where many points are connected to a small number of hubs. At the high school, on the contrary, there was no core group. Instead, the romantic and sexual network at the school created long chains of connections that spread out through the community, with few places where students directly shared the same partners with each other. This can be comparable to rural phone lines that run from a long main line to individual houses. One single component of the network was found to have 288 linked students, in one long chain. This chain featured 52% 52, percent (288) of the romantically involved students at the high school, but most students had only one partner and they had little idea of their connections to the long network chain. Adapted from Bearman, P.S., Moody, J., & and Stovel, K. (2004). Chains of Affection: The Structure of Adolescent Romantic and Sexual Networks. American Journal of Sociology. 110 (1), 44-91. 11. Which of the four models of disease infection network structure supports the results of the adolescent sexual networks study? Please choose from one of the following options. 1 Panel A 2 Panel B 3 Panel C 4 Panel D 12. A study shows that a central group of infected people such as sex workers transmits disease out to others but does not pass infection directly among themselves. Which of the four models of disease infection network structure demonstrates this finding? Please choose from one of the following options. 1 Panel A 2 Panel B 3 Panel C 4 Panel D

148


MCAT Practice Test 1.pages

13. Which of the following statements best explains Panel C? Please choose from one of the following options. 1 It is a structure with few cycles, low redundancy, and sparse density. 2 One individual’s past partner is tied through multiple chains to his or her current or future partner. 3 Two populations of persons engaged in different behaviors are linked by one actor. 4 One actor shows the ability to stop the transmission of STDs between two networks. 14. For epidemiological studies exploring flu transmission, “random mixing” captures the essential aspects of the diffusion process, for example, you do not choose to sit next to a person with the flu. However, for STD transmission studies, a crucial aspect to consider would be “preferred mixing” where people select partners with similar characteristics. Which of the following concept is similar to “preferred mixing”? Please choose from one of the following options. 1 Labeling 2 Symbolic interactionism 3 Stigma 4 Homophily 15. Based on the results of the study, which of the following makes sense as a policy intervention to prevent the spread of STDs in teenage populations? Please choose from one of the following options. 1 We should target high-risk teenagers to help curb the overall spread of STDs. 2 We should have comprehensive STD education for all adolescents, not just those at highest risk. 3 We should isolate teenagers who already have STDs to prevent increased incidence of new infections. 4 We should target sexually active teenagers who are the main core conduits of STDs.

149


MCAT Practice Test 1.pages

Questions 16. 17. 18. 19. 20. are NOT based on a passage. 16. Clinicians will see large amounts of data (labs results, patient symptoms, etc) over the course of their practice, and their brains will subconsciously group that data along certain established principles. Awareness of this underlying mechanism will help clinicians identify potential bias and provide better care. Which of the following best explains the Gestalt principles of grouping? Please choose from one of the following options. 1 The whole is more than the sum of its parts 2 Invariance, reification, and multistability are necessary to understanding perception 3 The mind processes the whole rather than the sum of its parts 4 Reality is reduced to its simplest form 17. The release of which neurotransmitter is associated with an increase in alertness? Please choose from one of the following options. 1 Norepinephrine 2 Beta-endorphin 3 GABA 4 Serotonin 18. Which of the following disorders from the DSM 5 is often characterized by delayed language development and unusual communication patterns? Please choose from one of the following options. 1 Autism spectrum disorder 2 Selective mutism 3 Developmental coordination disorder 4 Specific learning disorder, dyslexia

150


MCAT Practice Test 1.pages

19. When a person is surprised by a loud noise, the information from the cochlea travels to the auditory cortex. The startle reflex causes physiological changes which include the neck muscles tensing within two-tenths of a second from the stimulus to the increased tension in the neck muscles. What is the pathway that allows this extremely quick transmission of threat information? Please choose from one of the following options. 1 Auditory cortex => lateral and basolateral amygdala => midbrain => pons 2 Auditory cortex => midbrain => lateral and basolateral amygdala => pons 3 Auditory cortex => lateral and basolateral amygdala => pons => motor cortex 4 Auditory cortex => midbrain => pons => lateral and basolateral amygdala

20. Which of these questions would an individual ask during the secondary appraisal according to Lazarus and Folkman’s Cognitive appraisal model? Please choose from one of the following options. 1 Is this encounter a threat to myself or a loved one? 2 Do I have the resources to deal with this? 3 What are the possible benefits to my self-esteem from this encounter? 4 How might I be challenged by this situation?

151


MCAT Practice Test 1.pages

In studies of drug usage, individual physical or psychological characteristics have been used to differentiate users from non-users. This approach, common in “deviant behavior” studies, assumes that certain traits of individuals predispose or motivate them to engage in negative behavior. However, some researchers have suggested that drug usage behavior should be instead viewed as interactive social processes, and that the influence of peers should be taken into account. An early study (Study 1) focusing on marijuana users found that socialization is an important aspect in the maintained “pleasurable” usage of the drug (regular users). The researcher conducted participant observation and interviews with 50 marijuana users. Respondents who eventually used marijuana regularly were observed to undergo three stages: 1) they learn to smoke it in a way that produces real effects; 2) they learn to recognize the effects and connect them with drug use; and 3) they learn to enjoy the sensation they perceive. Individuals who do not go through the sequence of changes do not end up using marijuana regularly. In a more recent study (Study 2) illustrating the social aspects of drugs, researchers used data from National Longitudinal Study of Adolescent Health (Add Health) to further examine the peer effects of marijuana usage among adolescents grades 7-12. The investigators looked at close friends and classmates. The measure for close friends is “nominated peers”, which refers to the percentage of friends who used marijuana in the past 30 days and were named by the respondent as friends, and the classmates measure is “grade-level peers”, which refers to the percentage of peers who used marijuana in the past 30 days in the same grade and school as the respondent. The main results are shown below in Table 1.

152


MCAT Practice Test 1.pages

Table 1: Increase in likelihood of individual marijuana usage (in%), with every 10% increase in nominated peers and grade-level peers Nominated Peers Grade-Level Peers Individual Usage Increase Observations (n)

2.8%$$ 6,733$$

$

4.4% 19,335

*The coefficients for both are significant (p-values= 0.000). The two groups are not mutually exclusive. Source: Adapted from Becker, H. S. (1953). Becoming a Marihuana User. American Journal of Sociology, 59(3), 235-242. Ali, M. M., Amialchuk, A., & Dwyer, D. S. (2011). The Social Contagious Effect of Marijuana Use among Adolescents. Plos One. 6(1), e16183 21. According to the above study, which concept is most relevant for explaining sustained marijuana usage? Please choose from one of the following options. 1 Antecedent predisposition 2 Social subjectivity 3 Deviance theory 4 Symbolic interactionism 22. Which conclusion is best supported by the findings in Table 1? Please choose from one of the following options. 1 Classmates influence the likelihood of nominated peers using marijuana by 4.4% 2 A 4.4% increase in grade-level peers leads to more than 10% increase in the likelihood of individual marijuana usage. 3 Individual marijuana usage leads to a 2.8% increase in close friends’ marijuana usage. 4 A 10% increase in close friends who use marijuana leads to a more than 2% increase in the likelihood of individual marijuana usage.

153


MCAT Practice Test 1.pages

23. Which of the following conclusion can be drawn from Study 2 Please choose from one of the following options. 1 Peer groups do not significantly affect individual marijuana usage. 2 Interventions targeting all students in a school may be more costeffective than targeting individual marijuana users. 3 Identifying individual marijuana users is an effective way to decrease drug consumption rates at school. 4 If a student expresses deviant behavior, the likelihood of individual marijuana usage will increase. 24. If we study heroin usage and addiction using a similar social process model similar to Study 1, which of the following statements is most likely to be a reasonable finding? Please choose from one of the following options. 1 Cognitive expectancies due to the rewarding drug effects of heroin significantly influence an individual’s drug usage behavior. 2 Heroin usage is learned but self-interpretation of usage is the key indicator of prolonged abuse. 3 Individuals who express deviant behavior in general are more likely to become heroin addicts. 4 The interpretations of bodily effects and self-perceptions of heroin usage may be socially organized. 25. The above study emphasizes the effects of social interaction on drug usage. One further aspect to consider is how an individual’s drug-related behavior may be further reinforced by the way others judge or classify him/her. A guiding concept for this aspect would likely be: Please choose from one of the following options. 1 Labeling 2 Discrimination 3 Negative reinforcement 4 Prejudice

154


MCAT Practice Test 1.pages

There are 6 universal emotions: happiness, sadness, anger, surprise, fear, and disgust; each can be identified by universally produced facial muscle movements. Culturally linked emotional expressions also exist, such as winking or raising one eyebrow. In addition, the triggers for the display of emotion are often linked to cultural factors, such as Western cultures’ propensity to display emotion more openly than Eastern cultures. Darwin believed facial muscles signaled emotion and since the configuration of the facial muscles is universal, facial expressions enhanced communication. This, in turn, led to an increased chance of survival. In 1971, Ekman and Friesen published a study which supported the Darwinian viewpoint. They visited a Neolithic, preliterate culture in New Guinea called the Fore people, which had been isolated from Western contact until 12 years earlier. Only subjects that met very specific criteria were recruited, amounting to less than 3% of the total Fore population. A judgment task was given through a translator, who was instructed that there were no correct answers for the task. The translator told a well-rehearsed story which is shown in Table 1. After the story, subjects were presented 3 pictures, each displaying a different emotion (1 correct, 2 distractors). These pictures had to be correctly identified by at least 70% of the literate participants of Western or Eastern ancestry in a previous study. The subject was asked to point to the picture presented, which displayed the emotion that the subject believed was being described. Table 1 displays universal emotions along with the corresponding story that was told to study subjects.

155


MCAT Practice Test 1.pages

Table 1 Universal emotion

Story told to subject

Happiness

Friends have come to visit and he or she is happy.

Sadness

A family member (such as a child) has died.

Anger

The subject is angry and about to fight.

Disgust

The subject is looking at something that smells bad.

Surprise

The subject is seeing something new or unexpected.

Fear

The village is empty, there are no weapons, and the subject is attacked.

The null hypothesis was that there are no differences between the subjects representing the Fore tribe and the subjects representing Western or Eastern culture. No statistically significant differences were found in identification of emotions between groups except in the discrimination of fear from surprise and sadness, when the emotion described in the story was fear. Often, the subject could not tell the gender of the person but was 156


MCAT Practice Test 1.pages

still able to correctly identify the picture displaying the correct facial expression. Table 2 displays the results of the Fore Tribe responses in identifying the correct photograph corresponding with the correct emotion in the story. Table 2 Emotion describe d in story

Emotion shown in correct photograp h

Emotion shown in the first distractor photograph

Emotion shown in the second distractor photograph

Happine ss

Happiness

Fear

Anger

86

Sadnes s

Sadness

Anger

Disgust

69

Anger

Anger

Surprise

Sadness

81

Disgust

Disgust

Surprise

Sadness

77

Surprise

Surprise

Happiness

Anger

65

Fear

Fear

Surprise

Sadness

28

% choosing correct photogra ph

Adapted from: Ekman, P., & Friesen, W. V. (1971). Constants Across Cultures in the Face and Emotion. Journal of Personality and Social Psychology, 11, 124-129.

157


MCAT Practice Test 1.pages

26. What bias between the Fore people and Westerners could produce confounding results with the usage of the fear story (Table 1)? Please choose from one of the following options. 1 The Fore people could not interpret which emotion was being elicited due to a lack of common language, while Western people have a common language, English. 2 The Fore people have many emotions that are very similar in their expression, while individual Westerners display widely differing emotions. 3 The Fore people had difficulty understanding fear, surprise, and sadness because they had not been able to learn these facial expressions through imitation. Westerners, however, have observed these facial expressions constantly. 4 The Fore people have different social constructs, taboos, and may interpret a situation differently when compared to Westerners. 27. What is meant by “No statistically significant differences were found in identification of emotions between groups except in the discrimination of fear from surprise and sadness?� Please choose from one of the following options. 1 The null hypothesis was confirmed. There are no significant differences between Western and Eastern groups of people in the expression of emotions, except between fear, surprise, and sadness. 2 The alternative hypothesis was confirmed. There were likely biological differences in facial structure which caused the problems with the interpretation between fear, surprise, and sadness. 3 The alternative hypothesis was confirmed. With a 1 in 3 chance, the subject had a better chance of choosing the correct photograph with happiness than with fear as the correct photograph. 4 The null hypothesis was confirmed. There are likely cultural differences in the interpretation of fear which contributed to the difficulty with the distractors surprise and sadness.

158


MCAT Practice Test 1.pages

28. During the 1950’s and 1960’s, the Fore people suffered from Kuru, a spongiform encephalopathy. Suppose that during the examination of brain tissue samples from tribe members who had died from natural causes, many samples had bilateral damage to the amygdala which was not related to Kuru. Which designation (Table 2) would most likely be affected by this discovery? Please choose from one of the following options. 1 The correct photograph was sadness, while the distractors were anger and disgust. 2 The correct photograph was surprise, while the distractors were happiness and anger. 3 The correct photograph was disgust, while the distractors were surprise and sadness. 4 The correct photograph was fear, while the distractors were surprise and sadness. 29. Which of these theories accurately describes the process through which emotion is perceived by the Fore people during Ekman and Friesen’s experiment? Please choose from one of the following options. 1 Lazarus theory 2 James-Lange theory 3 Cannon-Bard theory 4 Schachter-Singer theory 30. The task given to the Fore people was originally developed by John Frederick Dashiell in 1927 for use with young children. What advantage(s) does this task have which allows it to be applied to the Fore people? Please choose from one of the following options. 1 A perfect translation is necessary to ensure that the emotion described in the story is understood. 2 The translator does not have to speak the language of the Fore people. 3 This task does not require literacy or a verbal response from the subject. 4 The subject must remember what emotion is being described.

159


MCAT Practice Test 1.pages

Jeff has two cats, Whiskers and Tiger. Jeff wants to play with both of them using a laser pointer, but the cats respond very differently to playtime. Whiskers is an active cat and loves to chase the red dot, but he sometimes gets so excited by playtime that he becomes overly agitated and attacks Jeff. Tiger is very lazy - he loves to look out the window from his bed and lounge in the sun; he is almost never interested in playing with the laser pointer. Jeff wants both of his cats to be equally active and healthy, so he decides to use principles from behaviorism to teach his cats better play behavior. He has two goals - to increase Tiger’s playtime, and to decrease Whisker’s aggressive playtime. He uses a variety of different strategies with his cats: he tries giving each cats treats when they play nicely, taking away Tiger’s bed to increase playtime, putting the cats in a carrier when they do not play nicely, and loudly yelling “No!” if Whiskers becomes aggressive. Jeff notices that the cats respond well to the treats, so he decides to vary when and how Whiskers and Tiger receive their treats. Table 1 outlines the different schedules of reinforcement that Jeff tries with Whiskers and Tiger. Table 1 Schedule

Cat

Strategy

A

Tiger

1 Treat for every 3 minutes of play

B

Tiger

1 Treat after a random amount of time of play

C

Whiskers

1 Treat every time play is nonaggressive

D

Whiskers

1 Treat after a random number of nonaggressive plays

160


MCAT Practice Test 1.pages

31. One of Jeff’s friends suggests giving Whiskers a treat when he is aggressive, as a way to distract him. Could this be an effective way to teach Whiskers to behave properly during playtime? Please choose from one of the following options. 1 No, Whiskers would be negatively reinforced to act out more often 2 No, Whiskers would be positively reinforced to act out more often 3 Yes, Whiskers would be negatively reinforced to behave properly 4 Yes, Whiskers would be positively reinforced to behave properly 32. What type of reinforcement schedule does Schedule C describe? Please choose from one of the following options. 1 Variable interval 2 Fixed interval 3 Variable ratio 4 Fixed ratio 33. Which of the following describes why rewarding Tiger with a treat for every 3 minutes of play could be more effective in changing his behavior than taking away his bed? Please choose from one of the following options. 1 More immediate feedback following desired behavior makes for a stronger consequence 2 Negative reinforcement is extinguished more quickly than positive reinforcement 3 Punishment is not the best way to shape an animal's behavior 4 Variable reinforcement is learned more quickly than fixed reinforcement 34. When Jeff put the cats in a carrier when they didn't play nicely, what kind of operant conditioning strategy was he using? Please choose from one of the following options. 1 Positive reinforcement 2 Negative reinforcement 3 Positive punishment 4 Negative punishment

161


MCAT Practice Test 1.pages

35. Jeff has been successfully increasing the number of times Whiskers has to play nicely before he receives a treat, until finally he decides he can stop giving treats during playtime all together. Whiskers suddenly becomes ultra-aggressive - he bites, hisses, and scratches at an increased rate when playing with the laser pointer. Which of the following best describes Whisker's behavior? Please choose from one of the following options. 1 Whiskers only responds to continuous reinforcement. 2 Whiskers' behavior has been shaped to be aggressive. 3 Whisker's aggressive behavior has become generalized. 4 Whiskers is experiencing an extinction burst.

162


MCAT Practice Test 1.pages

Questions 36. 37. 38. 39. 40 are NOT based on a passage. 36. Which of the following hormones is a hunger stimulant? Please choose from one of the following options. 1 Ghrelin 2 Insulin 3 Leptin 4 Dopamine 37. Josephine is an American college student who spends her sophomore year studying abroad in France. Her appreciation of French culture is heavily influenced by her experiences with French cuisine. She feels that the French believe it is important for families to take pride in the quality of their food and how it is prepared. She believes that the culture benefits from families eating and enjoying their food together. She ultimately decides French culture is superior to American culture. What term describes Josephine’s beliefs? Please choose from one of the following options. 1 Ethnocentrism 2 Xenocentrism 3 Cultural relativism 4 Cultural imperialism 38. Dr. Walsh is interested in seeing whether symptoms of depression can be manipulated using principles of classical conditioning. For several weeks of an experiment, he gives a group of patients a sweetened soda that has a mood-enhancing drug in it and notices that symptoms of depression improve significantly. Then, he removes the drug from the beverage and notices that the symptoms are still improved when the patients consume the soda. Which of the following is the conditioned stimulus in Dr. Walsh’s experiment? Please choose from one of the following options. 1 The sweetened soda 2 The improved symptoms in response to the soda 3 The improved symptoms in response to the drug 4 The mood-enhancing drug 163


MCAT Practice Test 1.pages

39. Research shows that our behavior is affected by intentions. In addition to intentions, however, it is also found that whether we have the resources or capacity to carry out these intentions also influence our behavior. This additional factor can be referred to as: Please choose from one of the following options. 1 Subjective norms 2 Planned behavior 3 Perceived behavior control 4 Attitudes 40. Deviations from homeostasis can create an internal state of biological and psychological tension. What is this best term for this state? Please choose from one of the following options. 1 Drive 2 Regulation 3 Equilibrium 4 Dissonance

164


MCAT Practice Test 1.pages

Prosopagnosia is a neurological disorder characterized by the inability to recognize familiar people based on facial information alone. Prosopagnosia, often called facial blindness, can be acquired through lesion, stroke, head trauma, or manifested without any discernible cause. Other patients with prosopagnosia may have developmental prosopagnosia (DP), which is characterized by a lifelong deficit in facial recognition and cannot be traced to acquired brain damage. A patient with DP may not be aware of this deficit because they would form the ability to recognize people by other distinguishing features that are unrelated to their facial characteristics. For patients with DP, the ability to recognize objects can be completely unaffected or only slightly impaired, but they may struggle to recognize close family members, friends, or themselves. At one time, DP was believed to be an extremely rare disorder, with only 9 case studies conducted between 1947 and 2001. In recent years this belief has been challenged as increased numbers of patients are diagnosed with DP, and researchers now believe that as many as 1 in 50 people may have some form of prosopagnosia. A researcher interested in prosopagnosia conducts an Internet survey in which respondents click a link, sign into a Social Media site, and are connected to a test battery that shows them a series of photos of famous faces. Each face is isolated from any other identifying features and is presented with eyes forward, as shown in Figure 1, which shows the isolated face of Barack Obama, adapted from President Obama’s 2008 Official Presidential Portrait. As each photo appears, the subject is asked to provide the name they associate with the presented face. If the subject is not able to remember a name, they are told to provide a description of the famous person's work or experiences. Once the subject submits this response, the famous person's name is revealed and the subject is asked to self-report whether their response was correct or incorrect. If the name presented is unfamiliar, the subject selects a radio button labeled, “I do not know this person� and their response is not counted against their accuracy score. Afterwards, the researcher calculates the percentage of correct answers, excluding the pictures described as unfamiliar. The percentage of faces recognized by each respondent is presented in Figure 2 , and mean accuracy across all respondents is 82%. 165


MCAT Practice Test 1.pages

Figure 1

Figure 2

166


MCAT Practice Test 1.pages

41. Based on the information in the passage, which of these would describe the most accurate top-down processing that a prosopagnosic might use to recognize faces? Please choose from one of the following options. 1 The use of monocular cues such as lighting and shading to recognize the person. 2 The use of eye color, skin color, and height to recognize the person. 3 The interpretation of gait, tone of voice, and context to recognize the person. 4 The use of facial cues to recognize the person. 42. Which of the following brain areas could produce symptoms of prosopagnosia if damaged? Please choose from one of the following options. 1 Frontal lobe 2 Fusiform gyrus 3 Cerebellum 4 Wernicke's area 43. Which of these people are likely to be diagnosed with DP? Please choose from one of the following options. 1 A 45-year-old man who may have had head trauma early in life and has always had trouble with faces. 2 A 12-year-old boy who has a lesion on the occipital lobe and can no longer recognize shapes, objects, or people. 3 A 65-year-old woman who recently had a stroke and is having trouble naming faces. 4 A 31-year-old woman who has always had trouble with faces. 44. Which of these could be a possible reason for the extremely low score in the 6th respondent? Please choose from one of the following options. 1 The researcher failed to check whether the subject was colorblind. 2 The researcher chose faces that most people did not know. 3 The respondent did not know most of the faces presented. 4 The respondent purposefully responded incorrectly.

167


MCAT Practice Test 1.pages

45. The researcher decides to examine the results item by item, and finds that one famous face has been mis-identified by every participant - they all name her as a different actress! He realizes that the photo he chose makes the actress look like a different woman. In terms of psychometric properties, what does this item have? Please choose from one of the following options. 1 Reliability, but not validity. 2 Validity and reliability. 3 Neither validity nor reliability. 4 Validity but not reliability.

168


MCAT Practice Test 1.pages

Autism has very diverse symptoms, and comprises many similar neurological disorders, which are classified as autism spectrum disorders (ASD). Temple Grandin is an autistic woman who is best known for her research on animal behavior, pressure therapy, and the design of the ‘squeeze box’ (shown in Figure 1), a pressure device she designed after observing the calming reaction in cattle during immunization when they were confined in a squeeze chute. As a toddler Grandin would rock, spin, or become fixated on objects for hours. Grandin suffered from hypersensitivity to touch, sound, and many other types of stimuli. Her reaction was often an intense, spontaneous, and uncontrollable panic attack. A noticeable change in her anxiety and sensitivity level occurs after approximately 5 to 15 minutes in the squeeze box, with diminishing results after 45 minutes to an hour. The perceptions of pain and hypersensitivity are psychophysical. Although there is a physiological factor, this does not fully describe the experienced pain or hypersensitivity. An autism researcher conducted a study on the efficacy of Grandin’s squeeze box on children displaying signs of ASD. The child was shown how to operate the squeeze box; a trusted adult entered it and operated it, thereby demonstrating that the device was safe. The child was then asked to enter, was given control, and was asked to operate the machine for 15 minutes at different pressures throughout. Every 3 minutes the child’s stimulation was assessed. The data is shown in Figure 2 below. Figure 1: Temple Grandin’s squeeze box, front view. A female subject is inside with her head and hands protruding. Deep touch pressure is applied by the squeeze box in response to the subject’s manipulation of the joystick (increasing and decreasing pressure consistently across the entire body).

169


MCAT Practice Test 1.pages

Figure 2: Average Likert scores for all subjects. Likert scale ranging from 1 to 5 ( 1 being calm, 5 being extremely stimulated). Each child was treated with the squeeze box for 15 minutes.

170


MCAT Practice Test 1.pages

Adapted from: Grandin, T. (1992). Calming effects of deep touch pressure in patients with autistic disorder, college students, and animals. Journal of Child and Adolescent Psychopharmacology, 2(1), 63-72. 46. Which mechanoreceptors would likely fire when the squeeze box is first engaged and touching the child’s body and when pressure is released? Please choose from one of the following options. 1 Meissner corpuscle and Merkel receptor 2 Merkel receptor and Pacinian corpuscle 3 Merkel receptor and Ruffini cylinder 4 Meissner corpuscle and Pacinian corpuscle

171


MCAT Practice Test 1.pages

47. How does sensory adaptation impact the efficacy of the squeeze box? Please choose from one of the following options. 1 Sensory adaptation quiets the nerves allowing the subject to adjust to the constant pressure. 2 Sensory adaptation has little impact on hypersensitivity with the use of the squeeze box. 3 Sensory adaptation has no impact on hypersensitivity with use of the squeeze box since there is constant pressure. 4 Sensory adaptation allows the subject to become adjusted to the pressure, thereby lowering hypersensitivity. 48. During the test, some children reported feeling like they were floating or were unaware of the position of their body in space. Which of these senses controls the symptoms that these children experienced? Please choose from one of the following options. 1 Nociception 2 Mechanoception 3 Kinesthesia 4 Proprioception 49. What conclusion can be drawn from the graph in Figure 2? Please choose from one of the following options. 1 Adaptation is the main cause of the increase in perceived comfort over time. 2 The length of time spent in the squeeze box is not related to the perceived comfort over time. 3 The refractory period is a major influence on the amount of perceived comfort over time. 4 The refractory period and adaptation have little influence. Psychological comforting is perceived as time increases.

172


MCAT Practice Test 1.pages

50. How does the average child in this study react to treatment in the squeezebox? Please choose from one of the following options. 1 The average child was unaffected by the treatment. 2 The average child became increasingly more calm as the treatment progressed. 3 The average child did not respond for the first 6 minutes and became only slightly calmer by the end of treatment. 4 The average child became more stimulated throughout the treatment.

173


MCAT Practice Test 1.pages

Pavlov’s dogs are perhaps the most famous case of classical conditioning. In his experiments Pavlov paired the ringing of a bell with presenting his dogs with food, and then he measured the amount of saliva produced by ringing the bell alone. By taking these measurements before and after the pairing, it was shown how a stimulus that normally has no response could be paired with a stimulus that does. The principles of classical conditioning extend far beyond canines and saliva; they can even be used to condition our immune system as a means of treating autoimmune diseases. Lupus is one such autoimmune disease, and treatment requires the suppression of a person’s immune system to protect their tissues and organs from being targeted by their own white blood cells. Cyclophosphamide is a chemotherapeutic drug taken orally or intravenously that is considered to be the standard treatment for lupus; however, like many chemotherapies, the side-effects of the drug can be quite severe. In 1992 a team of researchers showed that the human immune system can be classically conditioned, such that an 11 year old girl suffering from lupus was able to have significant reduction of her symptoms without any need of immunosuppressant drugs. This provided an opportunity to treat their patient’s disease while avoiding the deleterious side effects of cyclophosphamide. For their experiment, the researchers utilized a “Compound CS” which was a liquid that tasted of cod liver oil and had the smell of a rose. They paired Compound CS with the cyclophosphamide treatments on 6 occasions for a year. Every other month of treatment, the researchers did not administer the cyclophosphamide, and simply gave their patient compound CS instead. Despite administering the immunosuppressant drug at half the normal dose, the 11 year old patient still evidenced immunosuppression and continued to do well after a 5 year followup.

174


MCAT Practice Test 1.pages

*Citation: Giang DW, Goodman AD, Schiffer RB, Mattson DH, Petrie M, Cohen N, Ader R. J Conditioning of cyclophosphamide-induced leukopenia in humans. * Neuropsychiatry Clin Neurosci. 1996 Spring;8(2):194-201. Cohen N, Ader R. Immunomodulation by classical conditioning. Adv Biochem Psychopharmacol. 1988;44:199-202. 51. If cyclophosphamide is a standard drug treatment for lupus, why should a physician consider using Compound CS and classically conditioning his or her patients? Please choose from one of the following options. 1 Classical conditioning could be used in cases where the drug is ineffective in treating lupus 2 Compound CS could be helpful when a drug’s side-effects are not tolerable 3 Compound CS could be helpful in ending the use of cyclophosphamide to treat lupus 4 Classical conditioning could be used when a patient is not compliant with treatment 52. In Pavlov’s experiment and in the 1992 experiment involving autoimmunity, the bell and Compound CS both represent which of the following aspects of Classical Conditioning? Please choose from one of the following options. 1 Unconditioned stimulus (US) 2 Conditioned response (CR) 3 Conditioned stimulus (CS) 4 Unconditioned response (UR)

175


MCAT Practice Test 1.pages

53. Why did the researchers choose Compound CS to be used as the conditioned stimulus? Please choose from one of the following options. 1 Compound CS could be used to help the patient have a positive treatment experience 2 Cod liver oil and rose perfume would not normally interfere with the immune system 3 Cod liver oil and rose perfume as a placebo would mimic the same methods of treatment as cyclophosphamide 4 Compound CS could lessen side-effects when taken in conjunction with cyclophosphamide 54. The use of Compound CS carries many ethical considerations because its use involves the replacement of a standard drug treatment with what is essentially a placebo. Which of the following would NOT support the use of Compound CS for a patient study? Please choose from one of the following options. 1 Administering Compound CS would be no different from administering placebos in other experimental studies 2 Obtaining consent from a patient before undergoing treatment with Compound CS would better justify their participation in the study 3 Giving Compound CS to a patient would not cause additional harm and therefore there would be no risk in replacing cyclophosphamide 4 Enrolling a patient in the study would be better justified if they were unable to tolerate the side effects of cyclophosphamide 55. If a dog was classically conditioned to salivate upon hearing a bell ring, which of the following terms would describe the salivation? Please choose from one of the following options. 1 Conditioned response (CR) 2 Conditioned stimulus (CS) 3 Unconditioned stimulus (US) 4 Unconditioned response (UR)

176


MCAT Practice Test 1.pages

Questions 56. 57. 58. 59. are NOT based on passages. 56. Which of the following statements accurately describes the terms “sex” and “gender?” Please choose from one of the following options. 1 Sex refers to biological identity, whereas gender refers to psychological or social identity. 2 Sex and gender are interchangeable terms; both describe biological and psychological or social identity. 3 Sex and gender are interchangeable terms; both describe choices regarding sexuality and romantic partnerships. 4 Sex refers to psychological or social identity, whereas gender refers to biological identity. 57. Which of these would describe a research question viewed from a microsociology perspective? Please choose from one of the following options. 1 How does the conflict between Republicans and Democrats in Congress affect the U.S. role in the Middle East? 2 How does the patenting of seeds for corn, soybeans and other crops by large businesses such as Monsanto cause a conflict between local farmers and agribusiness? 3 What impacts do cultural views of medicine have on the expectations of care provided? 4 What role do people who self-identify as locavores play in their community?

177


MCAT Practice Test 1.pages

58. Structural theories of stratification predict that groups with low positions in social hierarchies experience high rates of mental health problems. The “double jeopardy” or “triple jeopardy” hypotheses postulate that groups that are subordinate in multiple stratification systems such as gender, race, and class are especially high risk. These hypotheses best demonstrate which of the following concepts? Please choose from one of the following options. 1 Intersectionality 2 Cultural bias 3 Symbolic interactionism 4 Social constructionism 59. The “American Dream” is the idea that if a person believes in their ability to apply themselves and work towards their goals, they can achieve success and happiness. Which of the following qualities does the American Dream describe? Please choose from one of the following options. 1 Self efficacy 2 Self concept 3 Self esteem 4 Self awareness

178


Turn static files into dynamic content formats.

Create a flipbook
Issuu converts static files into: digital portfolios, online yearbooks, online catalogs, digital photo albums and more. Sign up and create your flipbook.